archerreview

Pataasin ang iyong marka sa homework at exams ngayon gamit ang Quizwiz!

The ICU nurse assesses a comatose patient with a known lesion to the medulla. Which breathing pattern would the nurse expect to assess? A. Cheyne-Stokes B. Apneustic breathing C. Central neurogenic hyperventilation D. Cluster breathing

Explanation Choice D is correct. Cluster breathing is associated with lesions of the medulla or lower pons. This breathing pattern is characterized by clusters of breaths with irregular pauses in between. Choice A is incorrect. Cheyne-Stokes is associated with bilateral hemispheric disease or metabolic brain dysfunction and commonly occurs at the end of life. This breathing pattern is associated with cycles of hyperventilation and apnea. Choice B is incorrect. Apneustic breathing is associated with lesions of the mid or lower pons. This breathing pattern is characterized by a prolonged inspiratory phase or pauses alternating with expiratory pauses. Choice C is incorrect. Central neurogenic hyperventilation is associated with lesions of the brainstem between the lower midbrain and upper pons. This breathing pattern is characterized by sustained, regular, rapid, and deep breathing.

The nurse is caring for a client who was recently diagnosed with human immunodeficiency virus (HIV). Which of the following statements, if made by the client, would indicate a knowledge deficit? Select all that apply. "I started researching ways to tell my family that I have AIDS." "I recently stopped sharing household utensils and towels." "I will need periodic blood tests to measure the amount of virus." "I will not be able to continue my job as a phlebotomist." "If I achieve undetectable viral load status, I won't be able to transmit the virus to others."

Explanation Choices A, B, and D are correct. All of these client statements are false and indicate a knowledge deficit. The nurse should correct the client and remind them they have HIV - not AIDS. AIDS is when the client is HIV positive and has either a CD4+ T-cell count of fewer than 200 cells/mm 3 (0.2 × 10 9 /L) or less than 14% (even if the total CD4+ count is above 200 cells/mm 3 [0.2 × 10 9 /L]) or an opportunistic infection. HIV and AIDS is not transmitted through household utensils, towels, or toilets. The client will be able to continue their job as a phlebotomist because standard precautions are utilized, which will inhibit the transmission of HIV. The client will not have to take any additional precautions. Choices C and E are incorrect. All of these client statements are true. Periodic blood tests to measure the amount of virus (viral load) and their CD4 count are done to determine the efficacy of the prescribed treatment and the client's clinical trajectory. If the client were to achieve and sustain undetectable status, this signifies that the client is meeting the treatment goals. An undetectable status equals untransmittable to others. Additional Info ✓ Human immunodeficiency virus (HIV) is a retrovirus that may lead to Acquire Immunodeficiency Syndrome (AIDS) if untreated ✓ Modes of transmitting HIV include - • Sexual: genital, anal, or oral (low risk) sexual contact with exposure of mucous membranes to infected semen or vaginal secretions • Parenteral: sharing of needles ("sharps") or equipment contaminated with infected blood or receiving contaminated blood products • Perinatal: from the placenta, from contact with maternal blood and body fluids during birth, or from breast milk from an infected mother to child. Pregnancy is a risk factor for HIV as the pregnant individual has engaged in unprotected copulation. HIV testing is routinely done in the third trimester ✓ HIV is not transmitted by Casual contact in the home, school, or workplace Sharing household utensils, towels, linens, and toilet facilities Mosquitos or other insects do not spread HIV ✓ HIV and AIDS treatment goals include increasing the CD4 count and reducing the viral load (VL) to undetectable ✓ Everyone who has AIDS has an

The nurse is teaching a group of students about drug toxicity. The nurse is correct in stating which of the following? Select all that apply. "Naloxone is the treatment for opioid toxicity." "Magnesium is the treatment for lead toxicity." "N-acetylcysteine is the treatment for naproxen toxicity." "Calcium gluconate is the treatment for magnesium toxicity." "Flumazenil is the treatment for benzodiazepine toxicity."

Explanation Choices A, D, and E are correct. The antidote for opioid toxicity is naloxone which may be given IV, IM, Intranasal, or SubQ. Magnesium toxicity is treated with calcium gluconate. Flumazenil is indicated for benzodiazepine toxicity. Choices B and C are incorrect. High lead levels would be treated with succimer, which is indicated for heavy metals and lead. N-acetylcysteine is utilized in the treatment of acetaminophen toxicity - not naproxen which is an NSAID

Which interventions are appropriate for venous thromboembolism prophylaxis when caring for a non-ambulatory client? Select all that apply. Floating both of the heels using a pillow Apply sequential compression devices to the lower extremities Encourage range of motion exercises in the lower extremities Apply compression hose to the lower extremities Administer enoxaparin subcutaneously, as prescribed

Explanation Choices B, C, D, and E are correct. Sequential compression devices (SCDs) provide DVT prophylaxis by applying intermittent external pressure, pushing blood into deep veins, reducing stasis, and improving venous return. Range of motion exercises should be encouraged for a non-ambulatory client to encourage venous return. Thromboembolic deterrent (TED) hose promotes venous blood flow, prevents venous dilation, improves venous valve function, and stimulates endothelial fibrinolytic activity. Enoxaparin is a low molecular weight-based heparin given subcutaneously in the abdomen. This is a form of chemical VTE (venous thromboembolism) prophylaxis. Choice A is incorrect. Floating the heels is a measure to prevent a pressure ulcer. The heels may be floated using pillows or a specialty mattress that relieves the pressure. This would not be a measure to prevent a VTE. NCLEX Category: Reduction of Risk Potential Related Content: Potential for Alteration in Body Systems Question Type: Application Additional Info Prophylaxis for VTE includes mechanical and chemical agents. Mechanical prophylaxis involves SCDs and TED hose. Chemical prophylaxis includes medications such as enoxaparin or heparin. Risk factors for VTE include • Active cancer • Previous VTE (excluding superficial vein thrombosis) • Reduced mobility • Known thrombophilic condition • Recent (≤1 month) trauma and/or surgery • Older adult (≥70 years) • Cardiac and/or respiratory failure • Acute MI and/or ischemic stroke • Acute infection and/or rheumatologic disorder • Obesity (body mass index [BMI] ≥30) • Ongoing hormonal treatment

The nurse receives a prescription from the primary healthcare provider (PHCP) for metoprolol 5 mL intravenous (IV) push x 1 dose. The nurse should take which priority action before administering the medication? A. Clarify the prescription with the primary healthcare provider (PHCP) B. Assess vital signs C. Obtain a 5 mL syringe D. Assess the client's allergies

Explanation Choice A is correct. This prescription is inaccurate and requires clarification with the PHCP before moving forward. This medication was prescribed as a volume of 5 mL, not the precise dosage amount to be administered (for example, it is okay to be prescribed 5 mg of metoprolol, not 5 mL). The nurse needs an accurate prescription that is complete before executing other steps in the medication administration process. Choices B, C, and D are incorrect. These actions are all correct to perform if the nurse has an accurate prescription. The prescription of metoprolol is incomplete because the actual dose was not provided. Metoprolol is a beta-blocker indicated in treating hypertension and specific dysrhythmias such as atrial fibrillation. Additional Info The nurse must clarify any inaccurate or incomplete prescriptions to maintain client safety. Components of a medication order include the name of the drug, dosage, route, and frequency.

The nurse is teaching a group of students on fluid and electrolytes. It would be correct for the student to identify which intravenous (IV) solution as hypertonic? Select all that apply. 3% saline. Dextrose 10% in water (D10W) 5% Dextrose with 0.45% Sodium Chloride Lactated Ringers (LR) 0.45% Sodium Chloride (0.45% NaCl)

Explanation Choices A, B, and C are correct. 3% saline, D10W (dextrose 10% in water), and D5W0.45% NaCl (5% dextrose with 0.45% sodium chloride combined) are all hypertonic solutions. Choice D is incorrect. Lactated Ringers (LR) is an isotonic solution. Choice E is incorrect. 0.45% sodium chloride (0.45% NaCl) is a hypotonic solution. Additional Info ✓ Isotonic solutions are utilized for fluid resuscitation for hemorrhaging and sepsis, severe vomiting, diarrhea, GI suctioning losses, wound drainage, mild hyponatremia, or blood transfusions (0.9% saline only) ✓ Isotonic solutions include 0.9% saline and lactated ringers (LR) ✓ Hypotonic solutions are utilized for intracellular dehydration and hypernatremia ✓ Hypotonic solutions include: 0.45% Sodium Chloride (0.45% NaCl) 5% Dextrose in Water (D5W) - D5W may be referred to as both isotonic and hypotonic based on the context. Please note that D5W enters the body as isotonic and quickly becomes hypotonic as the liver rapidly metabolizes glucose. Thus, D5W is a hypotonic solution inside the body. ✓ Hypertonic solutions are utilized for severe hyponatremia and cerebral edema. They should always be given via an intravenous pump ✓ Hypertonic solutions include 5% Dextrose in 0.9% Normal Saline, 3% saline, 5% Dextrose and 0.45% Sodium Chloride, and total parenteral nutrition (TPN)

The client presents for a routine antepartum visit Primary Healthcare Provider (PHCP) Progress Note A 40-year-old nulliparous female is at 25 gestational weeks. The client is receiving appropriate antepartum care, taking a prenatal vitamin, and is appropriately vaccinated. She has a pre-pregnancy medical history of vitamin D deficiency, metabolic syndrome, and hypothyroidism. Age: 40 years old Gestational Weeks: 25 Height: 5 feet, 7 inches Weight: 194 pounds (88 kilograms) Body Mass Index (BMI): 30 Fetal Heart Rate: 130 beats-per-minute Vital Signs Oral temperature 97° F (36° C); Pulse 85 bpm; Respirations 16; BP 119/73 mm Hg; O2 saturation 96% on room air. The client presents for a routine antepartum visit. After reviewing the PHCP's progress note and the nursing assessment, which antepartum testing would the PHCP likely prescribe? Complete the following sentence by dragging one (1) antepartum test and two (2) relevant nursing assessment findings into each blank. The client will need a 1____________________as evidenced by 2 -------- 1. 2 Antepartum Test Assessment findi

Explanation glucose tolerance test body mass index (BMI) of 30 advanced maternal age. The client has a pre-pregnancy medical history of metabolic syndrome, characterized by several metabolic derangements such as abdominal obesity and high fasting blood glucose. This pre-pregnancy medical history is significant for the development of gestational diabetes along with the client's BMI of 30. Another significant risk factor for gestational diabetes is the client's age of 40. Advanced maternal age is a risk factor for gestational diabetes. The client is at 25 gestational weeks, and considering the risk factors, a prescription for a glucose tolerance test would be anticipated. The fetal heart rate is normal (110-160 is a normal finding). The client's blood pressure is also normotensive, thus, not raising the suspicion for gestational diabetes. The client is at 25 gestational weeks and too early to have a non-stress or contraction stress test. Chorionic villus sampling is a test that may be performed to look for chromosomal abnormalities as early as 10 gestational weeks. NST can be performed as early as 26 weeks. A CST can be performed as early as 34 gestational weeks. Additional Info Risk factors for gestational diabetes include - Overweight (BMI 25-25.9) or obesity (BMI 30 or greater) Maternal age older than 25 Advanced maternal age Gestational diabetes in a previous pregnancy History of polycystic ovary syndrome History of prediabetes First-degree relative with diabetes A glucose tolerance test may be administered between 24 and 28 weeks of gestation if necessary.

While working in a pediatric cardiac intensive care unit, you are caring for a child diagnosed with tetralogy of Fallot. Upon entering the room for your first assessment in the morning, you find the child crying, cyanotic, and tachycardic. You recognize this as a hyper-cyanotic tet spell. Place the following actions in order of priority: Document the event Administer 100% oxygen Administer morphine sulfate Administer an IV fluid bolus Place the infant in the knees to chest position

Explanation Correct Answer is: Place the infant in the knees to chest position Administer 100% oxygen Administer morphine sulfate Administer an IV fluid bolus Document the event Correct answer: The priority in a hyper-cyanotic tet spell is to place the child in the knee-to-chest position. Tet spells occur when the infant with tetralogy of Fallot becomes acutely cyanotic due to infundibular spasm usually associated with feeding or crying. When this spasm occurs, there is decreased flow from the right ventricle due to the obstruction, resulting in severe hypoxia. Putting the child in the knee-chest position increases the intrathoracic pressure and blood flow to the lungs, therefore increasing oxygenation to body tissues. The following priority action is to administer 100% oxygen to meet the child's oxygenation requirements and relieve the hypoxia quickly. The next priority action is to administer morphine sulfate. This is the drug of choice for tet spells because it helps to calm the child down while simultaneously reducing the infundibular spasm that causes right ventricular outflow obstruction and, therefore, the hypercyanotic tet spell. The next nursing action is to administer an IV fluid bolus. This increases preload and, consequently, cardiac output, helping to increase perfusion and oxygenation to the tissues. Lastly, the nurse should document the event, actions taken, and the patient's response.

The nurse is caring for a client in active labor. Nurses' Note 23-year-old primipara at 39 gestational weeks was admitted for induction via oxytocin. Currently, she is 100% effaced and 10 cm dilated. An internal fetal spiral electrode and intrauterine pressure catheter were placed. Uterine contractions are now 2 to 2.5 minutes apart, 70 to 90 seconds in duration. The fetal heart tracing showed decreased fetal heart rate following uterine contraction. This pattern was present in more than 50% of the uterine contractions. Medications Oxytocin via continuous infusion Complete the following sentence from the list of options Based on the fetal heart rate tracing, the client is experiencing A................................that is caused by B.............................................. A Late deceleration, early deceleration, variable deceleration. B. reduced blood flow to the placenta, Umbilical cord compression. Fetal head compression

A= Late deceleration B= reduced blood flow to the placenta

The nurse is caring for a client who was recently admitted to the cardiac floor for angina. This client states that their chest pain occurs at the same time every day at rest. The client does not believe there are any precipitating factors. Which of the following types of angina is this client most likely experiencing? A. Variant angina B. Stable angina C. Unstable angina D. Nonanginal pain

Explanation Choice A is correct. Variant angina, also known as Prinzmetal's angina, occurs at about the same time every day, usually at rest. Variant angina is treated with calcium channel blockers. Choice B is incorrect. Stable angina occurs after activity and is relieved by nitroglycerin tablets. Choice C is incorrect. Unstable angina is less predictable and may precipitate myocardial infarction. Choice D is incorrect. This type of discomfort does not describe nonanginal pain. Additional Info ✓ Prinzmetal's angina occurs when there is a temporary spasm or constriction of one or more coronary arteries, leading to a sudden decrease in blood flow to the heart muscle. These spasms can occur even in the absence of significant coronary artery blockages. ✓ During an episode of Prinzmetal's angina, transient changes may be seen on an electrocardiogram (ECG). These changes can include ST-segment elevation, often resembling a heart attack, which usually resolves once the spasm is relieved. ✓ Other types of angina include Stable angina Unstable angina Microvascular Angina Silent Angina Postprandial Angina Nocturnal Angina

The nurse reviews a client's laboratory data. Which laboratory data requires follow-up? See the image below. Select all that apply. Laboratory Result Reference range Sodium 130mRq/L 135-145 mEq/L Potassium. 3.7 mEq/L 3.5-5.0 mEq/L Calcium 8.3 mg/dL 9.0-10.5 mg/dL BUN 12mg/dL 10-20 mg/dL Creatinine 1.0 mg/dL 0.6-1.2 mg/dL Sodium Potassium Calcium BUN Creatinine

Explanation Choices A and C are correct. These laboratory values require follow-up because they are not within normal limits. The labs depict hyponatremia (any sodium less than 135 mEq/L) and hypocalcemia (any calcium less than 9 mg/dL). Causes of hyponatremia include dehydration, diuretics (especially thiazides), and SIADH. Causes of hypocalcemia include hypoparathyroidism, chronic renal failure, and vitamin D deficiency. Choices B, D, and E are incorrect. These laboratory values are within normal limits and do not require follow-up by the nurse. Normal potassium 3.5-5.0 mEq/L Normal BUN 10-20 mg/dL Normal Creatinine 0.6-1.2 mg/dL

The home health nurse is discussing the administration of insulin glargine with a client. It would require follow-up if the client states that insulin glargine must be administered Select all that apply. only in the abdomen. using a 6 mm and 31-gauge insulin syringe. only with a meal. after rolling the vial of insulin. within 28 days of opening the vial.

Explanation Choice A is correct. This statement requires follow-up because insulin glargine is only administered subcutaneously. Subcutaneous injections do not have to be administered just in the abdomen as the client may self-administer in the upper outer thighs. Choice C is correct. This statement requires follow-up because insulin glargine does not need to be administered with a meal because this insulin has no peak. Therefore, the likelihood of hypoglycemia is extremely low. Corrective insulin, such as rapid and short-acting, is administered with a meal. Choice D is correct. This statement requires follow-up because glargine insulin does not require to be rolled before administration. For a client receiving NPH insulin, that vial must be rolled to resuspend it so that when it is aspirated into the syringe, the client doesn't risk hypoglycemia because of the variable pharmacodynamic and pharmacokinetic profile. By rolling the vial prior to aspirating the insulin, the medication will be stable and reduce the client's risk for hypoglycemia. Choice B is incorrect. This statement is factual as insulin syringe needles are measured in 28, 29, 30, and 31 gauges and lengths of 6 mm, 8 mm, and 12.7 mm. Choice E is incorrect. This statement is factual, as insulin has a shelf life of 28 days. Any unused insulin should be discarded after 28 days. Additional Info ✓ Insulin glargine may be administered with or without meals ✓ Insulin glargine has no peak, so the risk of hypoglycemia is negligible ✓ This insulin is used as a basal insulin to provide coverage throughout the day ✓ Insulin glargine should not be diluted or mixed with any other insulin

While auscultating a client's bowel sounds, the nurse notes a swooshing sound to the left of the umbilical area. What would be the nurse's priority action? A. Percuss over the area to assess for dullness B. Notify the primary healthcare provider (PHCP) C. Gently palpate the abdomen to assess for tenderness D. Ask the client about recent bowel movements

Explanation Choice B is correct. Upon auscultation, the nurse should suspect this client is presenting with an abdominal aortic aneurysm (AAA) due to the bruit or swooshing sound. The nurse should immediately notify the client's healthcare provider of this urgent situation. An AAA rupture can occur spontaneously or with trauma. If the aneurysm bursts, it may cause life-threatening bleeding. The aneurysm should be assessed immediately to determine the need for surgical intervention. Choices A, C, and D are incorrect. If a bruit is heard upon auscultation of the abdominal aorta, the nurse should suspect an aneurysm and not perform percussion or palpation due to the risk of rupture. Asking the client about bowel movements (choice D) would be appropriate for assessing the client's gastrointestinal system but would not be the highest priority for a suspected aneurysm. Additional Info Assessment findings concerning an abdominal aortic aneurysm (AAA) include a bruit in the abdominal region. The PHCP should order a computed tomography (CT) scan to verify the presence of an AAA. If the client has had a rupture, manifestations include gnawing-like back (or flank) pain that may radiate to the groin or buttocks.

The nurse is triaging clients who were involved in a bus accident. Which client should be prioritized for transport to the local trauma center? A client who A. has pain and significant swelling in the right forearm with an intact distal pulse and sensation. B. has profuse bleeding from a chest laceration and is experiencing apnea. C. has a crushed leg reporting no sensation and has no distal pulse. D. is experiencing severe anxiety and has abrasions on both arms.

Explanation Choice C is correct. Because of their compromised circulation, this client would be red-tagged using the emergency triage tagging system (red, yellow, green, and black). Red tags require emergent care because of an immediate threat to their life. This client has a crushed leg with no distal sensation or pulse, significantly threatening their circulation. Thus, this client is prioritized for immediate evacuation to the nearest trauma center as a red tag. Choice A is incorrect. Pain and swelling are expected with a potential fracture. This client is a green tag because the distal pulse and sensation are present. Choice B is incorrect. The client who is experiencing apnea would be a black tag. A black tag is assigned when death has occurred or is imminent. An individual with apnea signifies death or impending death. Thus, the nurse should focus on the immediate (red tag) client with a crushed leg with no distal sensation or pulse. Green-tagged injuries include closed fractures, sprains, strains, abrasions, and contusions. Choice D is incorrect. The client experiencing severe anxiety and abrasions to the arms would be classified as a green tag. Nothing in this client's situation suggests a physiological injury that needs to be seen immediately. Additional Info Emergent (red tags) include life-threatening injuries, including obstruction to the airway, severe hemorrhage, or shock. Immediate treatment is necessary. Urgent (yellow tags) include alterations in blood glucose (hypoglycemia), disorientation, and large wounds that need treatment within 30 minutes to 2 hours. Nonurgent (green tags) include minor injuries such as strains, sprains, simple fractures, or abrasions. Treatment may be delayed up to four hours.

A nurse is caring for a client receiving digoxin. The client's most recent digitalis level was 2.5 ng/mL. The nurse should take which action? Select all that apply. Withhold the client's scheduled dose. Administer the dose, as prescribed Assess the client's 24-hour urinary output Assess the client's most recent sodium level. Assess the client's heart rate and rhythm Obtain a prescription for an echocardiogram

Explanation Choices A and E are correct. The client's digitalis level of 2.5 ng/mL indicates toxicity. Digoxin has a narrow therapeutic index, which can cause significant side effects, such as cardiac arrhythmias (e.g., bradycardia, heart block, ventricular arrhythmias), even at plasma concentrations only twice the therapeutic plasma concentration range. Normal corrective serum digoxin levels range from 0.5-2 ng/mL. A level higher than 2 ng/mL is considered toxic. The nurse is correct in withholding the scheduled dose and assessing the client's heart rate and rhythm, as the client is likely to be experiencing bradycardia. Choices B, C, D, and F are incorrect. It would be incorrect to administer the next dose, as this would exacerbate the toxicity. Assessing the urinary output and sodium is not relative to digitalis toxicity and is not the priority here. A significant trigger in digitalis toxicity is hypokalemia, not hyponatremia. Notifying the physician regarding the toxic level is appropriate, but there is no reason to obtain an echocardiogram. An echocardiogram will not add any additional information at this point. Instead, an electrocardiogram must be obtained to look for any rhythm disturbances due to digoxin toxicity. Additional Info The normal therapeutic range for digitalis is 0.5-2 ng/dL. Hypokalemia is a significant cause of digitalis toxicity and may be induced by certain diuretics. The earliest manifestation of digoxin toxicity is a lack of appetite, nausea, and vomiting.

You are tasked with providing education to a 24-year-old newly diagnosed diabetic. The correct approach to providing this education is: Select all that apply. Assess the client's knowledge. Ask the client about their perception of barriers to controlling the disease. Suggest small behavior changes based on specific information. Provide "just-in-time" education at the time of discharge. Assess the client's preferred learning style

Explanation Choices A, B, C and E are correct. Educating the client and family is one of the primary functions of the registered nurse. The first step in this process is to assess the client's knowledge about the disease or condition. Is the information accurate and complete? If not, one of the goals in the process is to correct or supplement the knowledge. As the nurse talks with the client, it is essential to help the client identify what barriers they perceive to being able to control their disease. For example, a person with diabetes might think that they cannot afford the medications necessary to treat diabetes. If the client must give themselves injections, they might say that they won't be able to inject themselves due to fear. Once the barriers are identified, the nurse must develop a plan to overcome the obstacles. Often, this involves consulting with other team members. For example, a case manager might be called in to help with financial issues. A new diagnosis can be overwhelming. It is often helpful to suggest small behavior changes. For example, a diabetic client may indicate that they cannot change their eating habits. This might indicate to the nurse that a dietician consult would be helpful. The dietician may be able to suggest foods that can be substituted for favorite foods. Assessing the client's preferred learning style can help the registered nurse and other consults modify their educational strategies to best meet the needs of the client. Choice D is incorrect. To provide "just-in-time education" at the time of discharge is inappropriate. Education of the client must always begin at the time of admission, mainly if the treatment will require lifestyle changes. Part of the success of the therapy will be from the repetition of teaching and return demonstrations by the client and family. NCSBN Client Need Topic: Health Promotion and Maintenance, Sub-Topic: Health Promotion/Disease Prevention, Endocrine; Prioritization Additional Info Different learning styles can include: ✓ Visual - pictures, diagrams, videos ✓ Auditory - listening to instructions or information ✓ Reading - reading instructios or information ✓ Kinesthetic - active participation and hands-on learning With diabetes educati

You are assigned to take care of a client who just underwent a cholecystectomy. Which of the following would decrease the risk of developing atelectasis in this client? Select all that apply. Deep inspiration. Supine position with the head end of the bed elevated. Change position every 2 hours. Encourage the patient to cough at least 10 times/hr. Encourage use of incentive spirometry

Explanation Choices A, B, C, D and E are correct. Atelectasis is defined as the total or partial collapse of the alveoli. This is a common complication in the immediate postoperative period, especially after abdominal surgeries. If atelectasis is not addressed, it may progress to pneumonia. Since alveoli are responsible for gas exchange, alveolar collapse can lead to impaired gas exchange/impaired oxygenation. Post-operatively, the client may not be able to take deep breaths due to pain from the movement of abdominal muscles. This impaired expansion of the alveoli leads to the accumulation of secretions/mucus plug, decreased surfactant, as well as the obstruction of airway and collapse of alveoli. Additional factors that predispose to this may include hypoventilation, sedation, and reduced mobility. When such factors are identified, the nurse should encourage the client to adopt interventions to mitigate those factors and prevent atelectasis. Such interventions include: Encouraging clients to take deep inspirations (Choice A) and use incentive spirometry (Choice E). An incentive spirometer encourages the client to pursue deep breathing. Deep breathing aids in gas exchange and promotes the full expansion of the alveoli. Keeping the client in the supine position with the head end of the bed elevated (Choice B) or semi-recumbent area (head of the bed raised 30 to 45 degrees). This allows for maximum thoracic expansion by lowering the abdominal pressure on the diaphragm. Encouraging the client to change position at least every 2 hours (Choice C). This increases mobility and allows full chest expansion and increases perfusion to both lungs. Encouraging the client to cough at least ten times per hour (Choice D) when awake. This helps promote alveolar expansion. The above interventions are aimed at preventing atelectasis. However, the nurse should be aware of detecting atelectasis if it did end up happening. Physical exam findings assist in the diagnosis and include fever and decreased breath sounds on the side of atelectasis. In the case of complete atelectasis/collapse, the trachea/mediastinum may be shifted to the same side due to the pull by a collapsed lung. Atelectasis in the postoperative period is referred

The nurse is caring for a client in bilateral soft wrist restraints. The nurse should assess the client's Select all that apply. behavioral status. skin integrity. bowel sounds. neurovascular status. need for the restraint.

Explanation Choices A, B, D, and E are correct. Assessing a client in soft wrist restraints should occur at a minimum of every two hours (or per facility policy). Pertinent assessments should include the client's behavioral status, skin integrity, neurovascular status (pulses, capillary refill), and the continued need for the restraint. Choice C is incorrect. Assessing the client's bowel sounds is not pertinent related to restraints. This would be relevant if the client had a nasogastric tube (NGT) or an acute gastrointestinal problem. Additional Info Restraints should be used as a last resort if alternative methods are not effective. A nurse should never threaten a client with restraints. This is considered assault. The nurse may place a client who is violent in restraints without an order from the primary healthcare provider (PHCP). If this was to occur, the nurse has one hour to inform the provider and obtain an order. Restraints are never as needed (PRN). They should be discontinued at the earliest possible time. When restraining a client, the reason for the restraint must be explained to the client and the behavior the client needs to demonstrate for the restraints to be discontinued. The nurse should observe the client at frequent intervals to offer nutrition & toileting, assess their behavioral status, obtain vital signs, and provide range of motion. These intervals are determined by the facility and the type of restraint—the more restrictive the restraint and the younger the client, the more frequent assessment. Restraints must be able to quickly be removed via a quick release buckle (knots are no longer recommended). The nurses' documentation must be comprehensive, describing the reasoning for the restraints, alternatives utilized, the education provided to the client, the type of restraint utilized, how it was secured, and the ongoing behavior necessary to continue the restraint. The nurse should also document the intervals at which the restraints were released.

The nurse is teaching a client about newly prescribed insulin glargine. The nurse recognizes the need for further instruction when the client makes the following statement? Select all that apply. "I will take this insulin right before my meals." "I should roll this vial of insulin before removing it with the syringe." "This insulin will help control my glucose for 24 hours." "I can only inject this insulin into my abdomen." "I'm glad to know I can mix this with my regular insulin."

Explanation Choices A, B, D, and E are correct. These statements are incorrect and require follow-up. Insulin glargine is a long-acting insulin that has no peak effect. Thus, it is not taken with meals. It is dosed once a day to provide glucose control for 24 hours. Insulin glargine is not a suspension; thus, it does not need to be rolled like NPH. This insulin is not mixed with any other insulin. Insulin glargine does not have to only be injected into the abdomen. Choice C is incorrect. This statement is factual and does not require additional teaching. Insulin glargine provides basal glucose control for up to 24 hours. This, combined with a carbohydrate-controlled diet, should decrease the client's reliance on correctional insulin. additional Info Source : Archer Review Insulin glargine is a long-acting insulin that provides basal control of a client's glucose. This insulin is given daily and does not have to be given with meals. It is highly unlikely that the client would develop hypoglycemia from this insulin because it has no peak effect. Thus, it is usually safe to be given to a client who is a nothing-by-mouth (NPO) statu

The nurse is caring for a client who was prescribed carbidopa and levodopa for Parkinson's disease. The nurse should instruct the client that this medication may cause Select all that apply urine to appear darker. hallucinations. dizziness upon standing. dry, non-productive cough. painful rash that spreads and blisters.

Explanation Choices A, B, and C are correct. Levodopa-carbidopa is the mainstay treatment for Parkinson's disease. The medication helps with the movement symptoms but has an array of adverse effects, including psychiatric symptoms that may cause the client to experience nightmares, paranoia, sleep disturbances, and bouts of psychosis. This is because the medication increases dopamine levels causing these psychiatric symptoms. Orthostatic hypotension is a significant concern with this medication because of alterations in the baroreceptors. This effect is intensified if the client is on anti-hypertensives. Choices D and E are incorrect. A dry, non-productive cough is not associated with this medication. Instead, this would be found in clients prescribed an ACE inhibitor. Steven-Johnson syndrome is not related to this medication which is described as a painful rash that spreads and blisters. Additional Info Levodopa-carbidopa is a mainstay treatment for Parkinson's disease ➢ The nurse should educate the client to consume their dietary protein throughout the day to avoid fluctuations in levodopa absorption. A high-protein diet may reduce the effectiveness of the medication. ➢ This medication should be administered on time to avoid significant fluctuations in dopamine which may intensify symptoms. ➢ Darkening of secretions such as sweat, tears, and urine is common and harmless. ➢ Adverse effects include mood instability, paranoia, sleep alterations, orthostasis, and vomiting.

The quality improvement nurse plans an initiative to reduce risk factors for falls in the acute care environment. Which of the following risk factors should the nurse recommend be addressed? Select all that apply. inadequate client assessment communication failures dim lighting a client's medical history age of the client

Explanation Choices A, B, and C are correct. The nurse should recommend addressing inadequate assessment because this is a significant risk factor for falls that may lead to client injury. The nurse can recommend a standardized fall risk assessment tool and monitor its execution. Communication failures between staff are a significant contributor to falls. The nurse should recommend addressing this by standardizing the handoff report and placing signals outside a client's room (a particular light, fall risk bands, or pictures outside of the client's room). The nurse can plausibly recommend the repair of dim environmental lighting, which is a risk factor for falls, especially when the client is ambulating within their room. Choice D is incorrect. The nurse cannot plausibly recommend addressing a client's age or medical history in reducing falls in the acute care environment because the nurse cannot control these variables. Choice E is incorrect. While they inform the nurse of a client's risk (older age, medical history of dementia, etc.), they cannot be influenced by the nurse on the quality improvement panel, nor are they related to the environment. Additional Info ✓ Risk factors for falls Previous client fall A client with altered cognition Tubes or devices connected to the client Sensory deficit(s) (poor vision) Poor fitting footwear Poor staff communication Inadequate client assessment Medications such as benzodiazepines, anticholinergics, diuretics, antihypertensives, and opioids ✓ Falls are a significant concern in acute care settings, with research indicating that they are one of the most common adverse events that occur in hospitals. ✓ Falls in the acute care setting can lead to serious consequences for clients. These consequences can range from minor injuries, such as bruises, to more severe outcomes like fractures, head injuries, and prolonged hospital stays. ✓ Nurses need to be aware of medications that can affect a client's stability and alertness, as well as those that might interact negatively with one another.

The nurse is teaching a client about the newly prescribed medication, esomeprazole. Which statement, if made by the client, would require further teaching? Select all that apply. "I should take this medication with meals." "I should not take this with any other medication or food." "The medication will coat my ulcer so I can eat without pain." "I will need frequent laboratory tests while taking this medication." "I may need to take magnesium supplements while on this medication."

Explanation Choices A, C, and D are correct. These statements are incorrect and require follow-up. Esomeprazole is a proton pump inhibitor (PPI) in treating esophageal erosion, GERD, and peptic ulcer disease. The medication should be taken one hour before meals and with an ample amount of water. The medication does not fortify an existing ulcer, like sucralfate. The client does not require frequent laboratory testing while on this medication. Choices B and E are incorrect. Correct teaching for a client receiving esomeprazole would include taking the medication independent of any other food or medicine as it will decrease its absorption. PPIs have the proclivity of causing hypomagnesemia, and thus, magnesium supplementation may be recommended by the PHCP. Additional Info ✓ PPIs are the gold standard in the treatment of GERD. ✓ Medications in this class include esomeprazole, pantoprazole, and lansoprazole. ✓ The client should be instructed to take the medication first thing in the morning without food or other medications. ✓ The long-term use of a PPI has been linked to osteoporosis and hypomagnesemia. Therefore, it is reasonable to recommend weight-bearing exercises and magnesium and calcium supplements approved by the primary healthcare provider (PHCP).

You are working in a Family Practice office. A client comes into the office with right facial drooping. The physician makes a diagnosis of Bell's palsy. You know that the primary treatment for this disease is likely to include: Select all that apply. Surgery Prednisone Antibiotic Antivirals Enoxaparin

Explanation Choices B and D are correct. Prednisone or another corticosteroid is likely to be prescribed. The anti-inflammatory action of these medications may help to reduce the swelling of the facial nerve and lessen the impingement that is causing the facial drooping. Antivirals are controversial, but some studies show that the combination of antivirals with corticosteroids may be helpful in clients with severe facial drooping. Both of these medications should be given as soon as possible after the symptoms start. Physical therapy to massage facial muscles can help to minimize permanent damage. Choice A is incorrect. Although surgery was once a treatment for Bell's palsy, it is no longer recommended due to the risk of permanent nerve injury and hearing loss. Choice C is incorrect. Antibiotics provide no relief for this condition since it is not caused by a bacteria that will respond to medicine. Choice E is incorrect. Bell's palsy is not the same as a thrombolytic stroke, and therefore does not require heparin injections to thin the blood. NCSBN Client Need Topic: Pharmacological and Parenteral Therapies, Sub-Topic: Expected Actions/Outcomes, Neurologic

The nurse is teaching a patient who is scheduled for a thoracentesis. Which of the following information should the nurse include? Select all that apply. "This procedure will require you to receive general anesthesia." "You will need to report any shortness of breath following the procedure." "You will need to empty your bladder before this procedure." "After the procedure, a follow-up chest x-ray will be done." "You will need to be on a clear liquid diet one day before the procedure."

Explanation Choices B and D are correct. These two statements should be included in patient education about thoracentesis. A thoracentesis is a procedure indicated for pleural effusions. The client will need to report any dyspnea after the procedure (Choice B). Shortness of breath following the thoracentesis procedure may indicate either iatrogenic pneumothorax or re-expansion pulmonary edema. Pneumothorax is a common complication following thoracentesis (studies report post-thoracentesis pneumothorax rates ranging from 0 to 19%). The nurse should assess the client carefully for any signs of pneumothorax. Symptoms and signs of a pneumothorax include shortness of breath and reduced or absent breath sounds on the affected side. A more severe pneumothorax, such as tension pneumothorax, may present with obstructive shock. A nurse must notify the physician immediately if any of such signs/symptoms were to occur. A chest x-ray (Choice D) must be completed post-procedure to make sure there is no iatrogenic pneumothorax even if the patient did not show any of the above signs or symptoms. Re-expansion pulmonary edema (REPE) is a complication that occurs after rapid re-expansion of a collapsed lung within 1 to 24 hours. It has been reported <1% in most studies are associated with high mortality. The pathophysiologic mechanism of REPE is unknown. Clinical features vary from cough and chest tightness to acute respiratory failure. Treatment is usually supportive and includes continuous non-invasive positive pressure ventilation or mechanical ventilation in severe cases; some patients also require vasopressors, steroids, and diuretics. Choice A is incorrect. Thoracentesis is a bedside procedure and can be completed under local anesthesia. Choice C is incorrect. It would be inappropriate to advise that the client empty their bladder before the procedure. Choice E is incorrect. Finally, a clear liquid diet one day before the procedure would be appropriate for a colonoscopy - not for a Thoracentesis.

The nurse is teaching parents about antepartum testing. Which statements should the nurse include? Select all that apply. A. "Oral glucose tolerance testing will measure fetal activity at certain intervals." B. "A nonstress test may be used to measure fetal heart rate." C. "Amniocentesis may be used to assess if you have preeclampsia." D. "Chorionic villus sampling may be done to assess for neural tube defects." E. "You may need to fill up your bladder prior to an ultrasound."

Explanation Choices B and E are correct. A nonstress test is performed in the third trimester if the client has indications such as a high-risk pregnancy that may result in a stillbirth or complications such as fetal hypoxia. Ultrasounds typically require a full bladder as the fluid moves the uterus upward and assists with visualization. Choices A, C, and D are incorrect. Oral glucose tolerance testing is completed between 24-28 weeks of gestation. This test is used to determine if the client has gestational diabetes and does not take into account fetal activity. Amniocentesis is an antepartum test that may be used to determine the gender of the fetus, lung maturity, neural tube defects, or chromosomal abnormalities. Chorionic villus sampling is a test that may be performed as early as ten gestational weeks to determine if the fetus has any chromosomal abnormalities. Additional Info Amniocentesis is performed for a variety of indications and at different gestational ages. Common indications for amniocentesis include identifying chromosomal, metabolic, or genetic abnormalities. Amniocentesis can assist in determining fetal lung maturity (FLM) status. After this procedure, women should be instructed to report signs of bleeding, amniotic fluid that continues to leak after 24 hours, severe cramping that lasts several hours, or a temperature greater than 100.4°F

The registered nurse (RN) assigns client care to a licensed practical/vocational nurse (LPN/VN). Which of the following should the RN assign to the LPN? Select all that apply. A client requiring an assessment of their current medications A client needing a nasogastric tube (NGT) for enteral feedings A client with an insulin pump and is unsure of how to load the insulin A client with unstable blood pressure following adrenalectomy. A client requiring airborne isolation and bronchodilators via an inhaler

Explanation Choices B and E are correct. Skills such as the insertion of an NGT are within the scope of an LPN/VN. The RN can delegate this to the LPN/VN. Further, LPN/VN's may care for a client in isolation as well as administer bronchodilators via an inhaler. Practical/vocational nurses should get the most stable patient assignment. Choices A, C, and D are incorrect. Assessment of a client's medication regimen is the responsibility of the RN, as well as teaching a client who is unsure about their insulin pump. Finally, the LPN should be assigned the most stable client. A client following adrenalectomy may consequently develop unstable blood pressure; thus, this should be assigned to the RN.

The nurse is performing a physical assessment on a client with Cushing's disease. Which assessment findings should the nurse expect? Select all that apply. Hypotension Acne Hirsutism Buffalo hump Truncal obesity

Explanation Choices B, C, D, and E are correct. Acne is an expected symptom of Cushing's disease. This is due to increased sex hormones, such as testosterone, estrogen, and progesterone. Excessive levels of these hormones cause oily skin to build up, often leading to acne. Hirsutism is defined as excessive body hair in either men or women in places where the hair usually is absent, such as the chin or cheeks of the face. Hirsutism is caused by increased androgens, or male sex hormones such as testosterone, in the body. A buffalo hump is the classic sign of Cushing's disease. This refers to a lump of fat that develops between the shoulder blades on the top of the back. It is due to the excessive amount of glucocorticoids that clients with Cushing's disease have. Glucocorticoids cause the breakdown of fats; when there are too many, they can cause fat redistribution. This often leads to fat in odd places, such as a buffalo hump. This excessive fat storage causes an individual to develop truncal obesity. Choice A is incorrect. Hyper- not hypotension, would be expected in a client with Cushing's. This is due to too many mineralocorticoids, specifically aldosterone. With increased aldosterone levels, the body retains too much sodium and water. With increased fluid in the vasculature, the blood pressure rises, and the client is hypertensive. Hypotension would be expected in a client with Addison's disease, whose body has a decreased amount of steroids.

The nurse is caring for a client experiencing pain. What barriers would the nurse recognize that the client may have in terms of reporting pain? Select all that apply. A feeling that the nursing staff will not answer their call for complaints of pain. Not wanting to be viewed as a complainer or drug seeker. A cultural bias. An ethnic bias. Fears about incurring more healthcare costs.

Explanation Choices B, C, D, and E are correct. Barriers that clients may have in terms of them reporting pain to the nursing staff include: Fears revolving around addiction and dependence on pain medications Not wanting to be viewed as a complainer or drug seeker A cultural bias An ethnic bias Fears about incurring more healthcare costs Choice A is incorrect. Although some clients may have a feeling that the nursing staff will not answer their call bell for complaints of pain, this is not a client barrier to them reporting pain to the nursing staff; it is, however, a nursing barrier to effective pain management and control. Additional Info Pain is often not adequately addressed across most healthcare settings. Clients most at risk for inadequate treatment of pain include: Older adults, especially in nursing homes Clients with a history of substance use Clients with a language barrier Unrelieved pain can result in a prolonged stress response, physiological changes (such as increased heart rate, blood pressure, and oxygen demand), reduced GI motility, delayed healing and immune response, and increased risk for chronic pain issues. Clients need nurses to assess and intervene to manage their pain, and nurses should consider factors that may inhibit reporting of pain.

The nurse is caring for a 71-year-old female in the emergency department (ED) Item 1 of 1 Nurses' Note 1425: 71-year-old female arrives via EMS with a concern about a stroke. At approximately 1350 client was at lunch with her family and suddenly stopped talking and fell to the right side. The client was unable to speak or follow verbal commands on the scene. Vital signs on arrival: 98.7° F (37.1° C), P 88, RR 18, BP 182/96. The client can blink her eyes and cannot follow verbal commands or express words. She is instructed to move each extremity but does not make any movement. Pupils are equal, round, and reactive to light. Right-sided facial drooping was noted. The client has a medical history of osteoarthritis, hypertension, and atrial fibrillation. 1427: A stroke alert was initiated at this time, and the client was transported to radiology for a STAT CT scan. 1438: Computed tomography scan completed. Physician at bedside evaluating the client and the results. 1444: Physician gave a verbal order for alteplase 0.9 mg/kg intravenous (IV) infuse over sixty minutes with a 10% alteplase bolus dosage given over one minute Diagnostics Computed Tomography of Head WO Contrast: No evidence of c

Explanation For the client with an ischemic stroke that is prescribed alteplase, this thrombolytic puts the client at substantial risk for internal bleeding. This thrombolytic is dosed based on weight, and an accurate weight is necessary to ensure dose accuracy. The nurse should perform all invasive procedures before the initiation of alteplase. Two peripheral IVs are necessary because if one becomes non-patent, the nurse has a second that may be utilized. This prevents the need for venipuncture while the client is at high risk of bleeding. Baseline laboratory work needs to be obtained prior to the initiation of alteplase to ensure that the PT/INR is not already prolonged, which is a contraindication for alteplase therapy. Because the client will require frequent neurological assessments and how critical this client's condition is, the client should be admitted to critical care. This is standard when alteplase is prescribed. Discontinue the infusion if the patient reports severe headache or has severe hypertension, bleeding, nausea, and/or vomiting; notify the primary health care provider immediately. A client's blood pressure can be higher than normal during an ischemic stroke. This permissive hypertension calls for notification to the physician if the systolic blood pressure is 185 mm Hg or greater or diastolic is 110 mm Hg or greater during or after alteplase

The nurse is caring for a client with a major thermal burn. Which initial laboratory abnormalities does the nurse anticipate in response to the burn? Select all that apply. Hemodilution Hyperkalemia Metabolic Acidosis Hyperglycemia Hemoconcentration

Explanation Choices B, C, D, and E are correct. Following a major burn, significant fluid and electrolyte changes occur from cellular damage, which causes potassium to leak into the extracellular space. Thus, life-threatening hyperkalemia may occur. Metabolic acidosis is likely because of the impairment the burn causes to the kidney's ability to recycle bicarbonate. The discharge of catecholamines causes glucose release from the liver, raising the blood glucose. Finally, the loss of fluid causes hemoconcentration, illustrated by elevated hematocrit. Choice A is incorrect. Initially, the client with a major thermal burn will have hemoconcentration from all of the fluid loss. Hemodilution may occur later in the process from the fluid shift. Additional Info Source : Archer Review The criteria for a major thermal burn are as follows - ✓ Partial thickness burns greater than 10% TBSA ✓ Burns that involve the face, hands, feet, genitalia, perineum, or major joints ✓ Third-degree burns in any age-group ✓ Electrical burns, including lightning injury ✓ Chemical burns ✓ Inhalation injury

The nurse cares for a client with a potassium of 5.7 mEq/L(3.5-5 mEq/L). The nurse understands that this potassium level may be caused by Select all that apply. Cushing's disease. nasogastric tube suctioning. salt substitutes. hyperinsulinism. adrenal insufficiency

Explanation Choices C and E are correct. The client's high potassium level, 5.7 mEq/L is concerning. Salt substitutes contain potassium which makes them more palatable. Excessive intake may lead to hyperkalemia. Adrenal insufficiency causes hyperkalemia because of the insufficient amount of aldosterone, which causes potassium elimination. Less aldosterone, and less potassium elimination, equates to hyperkalemia. Choice A, B, and D are incorrect. Cushing's disease is likely to cause hypokalemia, not hyperkalemia. In this disease, the adrenal glands produce too much aldosterone. Aldosterone causes the body to excrete potassium, putting clients with Cushing's disease at risk for excessive potassium losses leading to hypokalemia. The client with an NG tube to continuous suction will likely experience hypokalemia, not hyperkalemia. NG tube suction removes all of the gastric contents, which are rich in potassium. With those excessive potassium losses, the client becomes hypokalemic. Hyperinsulinism is likely to experience hypokalemia, not hyperkalemia. Insulin facilitates the movement of glucose into cells. With it comes potassium, and therefore when there is too much insulin as there is in hyperinsulinism, too much potassium is moved into the cells, and the serum potassium level drops, causing hypokalemia. Additional Info The normal potassium level is 3.5 mEq/L to 5.0 mEq/L. Cardiovascular changes are the most severe problems from hyperkalemia and are the most common cause of death in patients with hyperkalemia. The nurse should obtain a 12-lead electrocardiogram and establish continuous telemetry monitoring. Treatment for hyperkalemia includes regular insulin, albuterol breathing treatments, and/or sodium polystyrene. Source: Archer Review

A nurse is assessing a pediatric patient with right-sided heart failure. Which of the following assessment findings should the nurse expect to observe? Select all that apply. Grunting Nasal flaring Ascites Hepatosplenomegaly Swellings of legs, ankles and feet

Explanation Choices C, D, and E are correct. Ascites is indicative of right-sided heart failure. This would be due to the right ventricle not pumping sufficient amounts of blood to the lungs; therefore, the blood backs up in the body, causing an increased amount of fluid in the interstitial space. Any signs or symptoms involving an increase in fluid status indicate right-sided heart failure (Choice C). Hepatosplenomegaly is indicative of right-sided heart failure. This would be due to the right ventricle not pumping sufficient amounts of blood to the lungs, and therefore blood backs up in the body, causing an increased amount of fluid in the liver and spleen, which leads to their enlargement. Any signs or symptoms involving increased fluid status would indicate right-sided heart failure (Choice D). Peripheral edema is a common sign of right-sided heart failure in pediatric patients (Choice E). Choice A is incorrect. Grunting is a sign of left-sided heart failure in an infant. It is a classic sign of respiratory distress in an infant. This serious finding should be reported to the health care provider immediately. Respiratory signs and symptoms indicate left-sided heart failure because the blood is backing up in the lungs due to the inability of the left ventricle to pump sufficient amounts out to the body. Choice B is incorrect. Nasal flaring is a sign of left-sided heart failure in an infant. It is a classic sign of respiratory distress in an infant. Respiratory signs and symptoms indicate left-sided heart failure because the blood is backing up in the lungs due to the inability of the left ventricle to pump sufficient amounts out to the body. Additional Info Manifestations of right-sided heart failure in children include ✓ Edema (swelling) in the feet, ankles, or legs ✓ Abdominal swelling or discomfort ✓ Enlarged liver ✓ Difficulty breathing, especially with exertion ✓ Fatigue or decreased activity tolerance ✓ Rapid heart rate ✓ Decreased urine output

The charge nurse is planning patient care assignments for a licensed practical/vocational nurse (LPN/VN). Which of the following would be an appropriate patient assignment for the LPN? Select all that apply. A 67-year-old one-hour post-procedure from a cardiac catheterization. An 88-year-old client who was just admitted for intractable pain secondary to metastatic cancer. A 42-year-old being discharged following a diagnosis of type 2 diabetes mellitus. A 75-year-old inpatient client with colon cancer needing colostomy care. A 50-year-old client being treated for herpes zoster with prescribed oral antivirals.

Explanation Choices D and E are correct. These are within the scope of LPN practice. LPNs can be assigned ostomy care and may administer oral medications to stable clients. A licensed practical nurse (LPN) can monitor and attend stable patients as long as they have been evaluated by a registered nurse (RN). As such an LPN can: Administer oral and parenteral medications. The LPN may not administer IV medications in some states. In the NCLEX exam, a student is better off not choosing that option. LPNs may reinforce patient teaching/education that was initially addressed by an RN. LPNs may perform focused assessments but only after the RN's initial estimate. The performance of the first assessment is outside the scope of the LPN. LPNs may attend routine procedures like bladder-catheterization, address tube feeding, and ostomy care. Choice C is incorrect. This client is being discharged and requires discharge teaching. LPNs can not be assigned initial discharge teaching, although they can reinforce patient education. Choice A is incorrect. This client requires close monitoring and is likely to be considered unstable, therefore they will need an RN. LPNs should be assigned patients who are predictable/stable but NOT immediate post-operative patients. Choice B is incorrect. This client was just admitted and has intractable pain. The LPN cannot be assigned a new admission. NCSBN Client need Topic: Management of care; Sub-Topic: Assignment, delegation, and supervision.

The nurse is caring for an older adult in the medical-surgical unit Health History 84-year-old female was admitted to the medical-surgical unit with a three-day history of abdominal pain, distention, nausea, and persistent vomiting. She reports that she has not had a bowel movement in five days and has no appetite. Vital Signs Oral temperature 101.1° F (38.3° C) Pulse 108/minute Respirations 22/minute Blood pressure 100/64 mm Hg Oxygen saturation 96% on room air. The nurse reviews the client's health history and vital signs Click to specify if the findings are consistent with a small bowel obstruction or appendicitis. Each row must have at least one but may have more than one response option selected Clinical Findings Small Bowel ObstructionAppendicitis Unable to pass stool Fever Distended abdomen Right lower quadrant abdominal pain Nausea and vomiting

Explanation Clinical features of a small bowel obstruction and appendicitis can be found in the below table Treatment for a small bowel obstruction involves operative or non-operative management. Nursing care involves establishing vascular access and rehydrating the client, as prescribed. Intravenous pain medication and nothing-by-mouth (NPO) status will likely be prescribed.

Nurses' Notes 1700: 73-year-old male reports explosive, watery, foul-smelling diarrhea that started two days ago. The client reports intermittent abdominal cramping that occurs with watery diarrhea. He says his wife made him come in to get medical attention because he was starting to 'feel weak' and 'probably dehydrated.' The client was assessed to have: a sunken eye appearance, dry, flaky skin, and thready peripheral pulses. VS: Oral Temperature 98° F (36.7° C), pulse 86/minute, respirations 16/minute, blood pressure 113/68 mm Hg, oxygen saturation 96% on room air. 1725: Stool sample of foul-smelling diarrhea sent to the lab. 1830: Laboratory result received. Physician notified of results Orders C. difficile stool PCR Current Medications Omeprazole 40 mg PO Daily Clindamycin 150 mg PO q 6 hours, Daily for seven days Lisinopril 10 mg PO Daily Multivitamin 1 tablet PO Daily Atorvastatin 60 mg PO Daily Laboratory RESULTS Ref. Renge C. difficile stool PCR + (+_-) The nurse reviews the nurses' notes, orders, current medications, and laboratory data. Based on the clinical data, select five (5) nursing interventions the nurse should implement. Obtain a prescription for metronida

Explanation It would be appropriate for the nurse to obtain a prescription for an antibiotic as C. diff is a gram-positive bacterium treated by vancomycin, metronidazole, or rifaximin. Clindamycin is an antibiotic with the highest risk of causing C. diff. Thus, the nurse needs to obtain an order to administer metronidazole. The approved antibiotics to treat C. diff include oral vancomycin, metronidazole, and rifaximin. Educating the client to wash surfaces with dilutional bleach and practice meticulous hand hygiene is essential in preventing reinfection. Alcohol is ineffective in eradicating the spore; thus, it would be appropriate for the alcohol sanitizers to be removed from the room. The client is showing manifestations of dehydration, and encouraging fluid intake is appropriate. Contact precautions should be implemented as droplets do not spread this pathogen. Cleansing enemas are not indicated in the treatment of C. diff. This intervention would dehydrate the client further. The client's age and current use of an antibiotic and proton pump inhibitor increase his risk of C. diff. Additional Info According to the Centers for Disease Control, the transmission of C. diff can be disrupted through: Meticulous hand hygiene with soap and water. Avoid using alcohol-based hand sanitizers. Using disposable healthcare equipment, such as blood pressure cuffs and stethoscopes. Disinfect surfaces with a bleach solution. Discontinuing unnecessary antibiotics.

The emergency department (ED) nurse is caring for a client who sustained a witnessed electrical burn. Triage Note 1730: 35-year-old male was brought to the emergency department (ED) by his father after they were working on electrical wiring at a residential house. The client's father witnessed his son grab a wire and sustain a significant 'jolt' for five to ten seconds. The client briefly lost consciousness and was disoriented immediately afterward. The client was immediately placed in the father's car and transported to the ED. A localized burn was noted on the client's right hand. Scant sanguineous drainage noted. The client reports pain of a '6' (0= no pain; 10= severe pain) that is worsened with movement. The client is alert and oriented to place and time; however, he does not recall the situation that brought him to the hospital. Glasgow Coma Scale (GCS) 14. The client reports that he feels like his 'heart is intermittently skipping.' Triage Vital Signs Oral Temperature 101 o F (38o C) Heart rate 94/minute, irregular Respirations 19/minute Blood pressure 115/69 mm Hg Oxygen saturation 95% on room air The nurse reviews the client's triage note Select one (1) condition and one (1)

Explanation The client sustained an electrical burn; an immediate concern associated with these types of burns is the client's cardiovascular status. The client's pulse is irregular, and this irregular pulse should prompt the nurse to immediately obtain a 12-lead electrocardiogram (ECG) and establish continuous telemetry monitoring. Electrical burns rarely result in carbon monoxide poisoning. Thus, this concern would be associated with a significant thermal burn with certain fumes. The client's GCS is 14 (normal is 15), and his incomplete orientation is likely due to the electrical burn that produced amnesia. This is common in certain electrical burns, especially at high voltage. While wound infection could be a concern if localized wound care is not provided, this is not the immediate concern. Additional Info ✓ Electrical burns may be caused by lightning or weapons such as tasers. ✓ Additionally, exposure to live power wires may cause this type of injury. ✓ The longer the electricity is in contact with the body, the greater the damage. ✓ It is a priority to terminate the electrical source (if possible) and then render care. ✓ The care that should be provided includes an immediate assessment of the client's cardiovascular status because of the chance of fatal dysrhythmias. ✓ Accidental electrical injuries may be avoided by refraining from inserting objects into an electrical plug. ✓ Electrical devices should not be used near a body of water. ✓ Any frayed electrical cords should be replaced.

The nurse is caring for a client in the first stage of labor Item 1 of 1 Nurses' Notes 1800: A 31-year-old primigravida at 38 gestational weeks 100% effaced and 2 cm dilated. Contractions occur every 15 minutes lasting for 20 seconds. The client reports that the contractions are becoming more painful and alternate between cramping and pressure. 1825: Informed consent was obtained by the physician for a planned epidural catheter with the administration of analgesia. Signature was witnessed by the RN. Continuous external fetal monitoring was applied. Normal variability was noted with the fetal heart rate of 110-130/minute. Vital Signs Temperature 98.0° F (37° C) Pulse 91/minute Respirations 16/minute Blood Pressure 138/76 mm Hg Oxygen saturation 95% on room air Drag (1) one condition and (1) assessment to fill in each blank of the following sentence Immediately following the placement of the epidural, the client is at the highest risk for---- and the nurse anticipates a prescription for----- Conditions hypertension infectionearly decelerations hypotension hyperglycemia Anticipated Prescriptions regular insulin magnesium sulfate 0.9% saline bolus broad-spectrum antibiotics internal

Explanation The most common adverse maternal effect associated with epidural analgesia is the development of maternal hypotension. This effect can be mitigated by infusing 0.9% saline bolus pre-procedurally. Alternatively, lactated ringers may be used instead of 0.9% saline. Infection may occur with an epidural, but this would not be seen immediately following its placement. Antibiotics are not routinely given before the initiation of epidural analgesia. This type of analgesia would not cause early fetal decelerations or maternal hyperglycemia. Additional Info For lumbar epidural analgesia, the common site selected is between L3 to L4. Essential monitoring parameters include the area for signs of infection, blood pressure (may cause hypotension), and urinary output (may cause urinary retention).

The nurse is caring for a client in the emergency department (ED) with an altered level of consciousness Item 1 of 1 History and Physical A 53-year-old male presented to the emergency department (ED) with his wife because the client had become quite tired over the past several days. Today, he was difficult to arouse and spoke incoherently. The client responds to his name during the assessment but does not respond to any other questions. His pulses were thready and slow. Obvious tenting was noted in the skin, which was warm and quite dry. No facial drooping was observed, and when asked to hold out his arms, he could not perform the task. In fact, he did not have many purposeful movements during the exam. The client has a medical history of gout, bipolar disorder, and hypothyroidism, for which he takes levothyroxine, allopurinol, and quetiapine. She reports that he has been taking his medications as prescribed. However, she noted he was recently placed on Prednisone 20 mg PO BID for a gout flare. He self-discontinued the drug after taking it for two weeks and feeling better, and he did not taper as directed. Vital Signs Temperature 98.0° F (37° C) Pulse 121/minute Respirations 16/minute

Explanation Adrenal crisis Cessation of prednisone This client exhibits signs and symptoms of an adrenal crisis because of the abrupt cessation of prednisone. Findings supporting the crisis include the client's lethargy, altered mental status, hypotension, tachycardia, and peaked T waves on the electrocardiogram. The significant dehydration associated with an adrenal crisis explains the client's hypotension and tachycardia. Their dry skin and tenting further support this. The peaked T waves on the ECG are evidence of hyperkalemia, a feature of an adrenal crisis. A myxedema coma is not likely because the abrupt cessation of the prednisone would trigger an adrenal crisis, not a myxedema coma. This coma is caused by the thyroid hormone being severely low. Finally, catatonia is a syndrome that may occur with bipolar disorder. Although the client does have some of the manifestations of catatonia (lack of purposeful movements, lack of speech [or incoherent speech]), this is not the case because of the hypotension, tachycardia, and peaked T-waves. Additional Info ✓ An adrenal crisis may be triggered by a sudden cessation of a corticosteroid (especially if it is a considerable dose and the duration is greater than two weeks). ✓ An adrenal crisis is a medical emergency manifested by hypovolemia which causes hypotension and tachycardia. ✓ Hypoglycemia is also a clinical feature along with hyponatremia. ✓ The client will have elevated potassium levels that may cause cardiac dysrhythmias. ✓ The priority treatment for a client with an adrenal crisis is prompt administration of intravenous hydrocortisone! ✓ Additional treatment includes intravenous fluids, glucose, and regular insulin to lower the potassium.

The nurse is caring for a newborn who just had a circumcision Item 1 of 1 Nurses' Note 1730 - First diaper change since circumcision. Three dried blood spots were noted in the diaper and a moderate amount of straw-colored urine. The diaper was changed, and petroleum jelly was reapplied to the penis. Erythema was noted at the tip of the penis. No crying or signs of pain distress were observed during the diaper change. Reviewed after-care instructions to the parents. Vital Signs Axillary Temperature 97.0o F (36o C) Pulse 134/minute Respirations 44/minute O2 saturation 96% on room air Click to specify if the statements made by the parent indicate effective teaching or require follow-up Statement Effective Teaching Requires Follow-up "I should gently wipe off any yellow exudate that may appear." "I can use premoistened alcohol-based baby wipes to decrease the risk of infection." "He should not ride in his car seat until the circumcision is fully healed." "I should fasten the diaper loosely while the circumcision heals."

Explanation After reviewing aftercare instructions with the parents of a newborn who just had a circumcision, the nurse should follow up with the parents if they state that they will wipe off any yellow exudate that may appear. This is a normal finding and will disappear within a week. Premoistened alcohol-based baby wipes should be avoided because they irritate the incision and cause discomfort to the infant. Infants should always ride in a rear-facing car seat, in the back seat, even after a circumcision. While the car seat may irritate, the infant's safety is essential, and riding anywhere else but a rear-facing car seat is dangerous. It would indicate effective teaching if the client states they will fasten the diaper loosely while the circumcision heals. This prevents the penis from adhering to the diaper and causing irritation. Additional Info Following circumcision, the nurse should educate the parent or guardian with the following aftercare instructions - Observe the circumcision site at each diaper change and check the amount of bleeding. Call the physician if more than a few drops of blood are present with diaper changes on the first day or any bleeding thereafter. Continue to apply petroleum jelly to the penis with each diaper change for the first 4 to 7 days or as directed by your pediatrician. If a PlastiBell ring is used, do not use petroleum jelly because it might make the ring fall off too soon. Keeping the circumcision site clean is important for healing. Squeeze warm water from a clean washcloth over the penis to wash it. Pat gently to dry the area. Fasten the diaper loosely to prevent rubbing or pressure on the incision site. Expect a yellow crust or scab to form over the circumcision site. This is a normal part of healing and should not be removed. The scab will fall off within 7 to 10 days

The nurse is caring for a client with an acute spinal cord injury. Which client finding would require immediate follow-up? Omitted A. absent bowel sounds B. blood pressure 134/82 mm Hg C. pulse 92/minute D. hyperreflexia

Explanation Choice A is correct. Absent bowel sounds, gastric distention, bradycardia, hypotension, and flaccid paralysis are concerning findings for spinal shock. When caring for a client following a spinal cord injury, spinal shock is one of the many complications which may occur within 48 hours following the injury. Choices B, C, and D are incorrect. This clinical data is not consistent with spinal shock. If spinal shock is suspected, the client will develop hypotension and bradycardia. This shock would depress reflexes, not cause hyperreflexia. Additional Info Spinal shock may occur immediately following a spinal cord injury or within 48 hours of the insult. The cause is thought to be the excessive amount of potassium in the extracellular space that reduces neural transmission. Manifestations associated with spinal shock include flaccid paralysis, absent bowel and bladder control, and loss of reflex activity. Males may develop priapism. Treatment involves frequent assessment of vital signs, correcting any fluid or electrolyte abnormalities, and prompt administering corticosteroids.

The nurse is educating a client about the inactivated influenza vaccine (IIV). The nurse should plan to teach the client that Select all that apply. the IIV effectively prevents influenza or decreases the disease's severity. pregnant women can receive this vaccine. you may receive this vaccine if you are allergic to penicillin. the IIV contains a live virus. the vaccine is administered to newborns following delivery.

Explanation Choice A is correct. Although the influenza vaccine will not prevent 100% of the cases, it will help prevent or decrease symptoms in 70 to 80% of cases. If the client does get influenza, the severity of the symptoms will be diminished thanks to the vaccine. Choice B is correct. Those who are pregnant may receive the inactivated influenza vaccine (IIV). It is the Live attenuated influenza vaccine (LAIV) that should not be administered to those who are pregnant. The LAIV is licensed for ages 2-49. Choice C is correct. Penicillin allergy is not a contraindication to the administration of the influenza vaccine. Very few contraindications exist to receiving the influenza vaccine. Some contraindications include children younger than six months of age, individuals with a previous severe reaction to a prior influenza vaccine, and a history of Guillain-Barré syndrome. Egg allergy is not a contraindication to this vaccine being administered. Choice D is incorrect. The IIV is inactivated and does not contain a live virus. IIV is administered to infants starting at six months. Thus, the infant's caregivers must get vaccinated to prevent transmission to the newborn. Additional Info ✓ The influenza vaccine may be administered to those six months and older. ✓ Egg allergy is not an absolute contraindication to vaccine administration. ✓ A history of Guillain-Barré syndrome within 6 weeks of receiving an influenza vaccine is another contraindication to all influenza vaccines. ✓ Influenza vaccine may be coadministered with another vaccine such as COVID-19. ✓ IIV and LAIV are available. LAIV (live attenuated influenza vaccine) is administered via nasal mist. ✓ The LAIV is recommended for individuals who are immunocompetent aged 2 through 49.

The occupational health nurse is teaching a group of unlicensed assistive personnel how to practice appropriate ergonomics. It would be appropriate for the nurse to recommend that Select all that apply. your feet are firmly on the floor while you are sitting in a chair. your feet are close together as you move or transfer a client. heavy objects be held far away from your body to achieve balance. you should squat to lift objects off of the ground. your neck should be extended as you look at the computer monitor.

Explanation Choice A is correct. Appropriate body mechanics are crucial to preventing injuries. Since most documentation is done electronically, ergonomics while using the computer is crucial. When sitting in a chair, the feet should be firmly on the ground to prevent stress on the lower back. The knees should be at the level of the hips or slightly lower. Choice D is correct. When picking objects off the ground, the individual should squat and not bend over. Squatting engages the thigh muscles to provide support as the object is lifted. Bending over would put stress on the lower back. Choice B is incorrect. A wide base of support (feet planted shoulder width apart) provides more stability versus the feet close together. As this wide base support is assumed, appropriate footwear is necessary so the individual does not lose balance when moving an individual (or object). Choice C is incorrect. The body's core is where objects should be held because this is the center of gravity. Holding objects with the arms out may put stress on the upper back. Choice E is incorrect. When working at the computer, the neck should be in a neutral position. Extension and flexion of the neck could cause cervical spine injuries. The shoulders should be relaxed. Additional Info Effective Measures to Prevent Back Injury Include ✓ Have the necessary assistance to move the object. ✓ Planning the move and communicating with the other individual who will assist you. ✓ Using the shoulder, upper arms, hips, and thighs as the predominant muscles to help with the move. ✓ Keep objects close to your body when lifting or carrying objects. ✓ Avoid twisting by using your feet to turn your body. Use a mechanical lift when necessary.

A client of Middle Eastern descent is currently 24 weeks gestation. Based on the client's culture, the client usually wears a long robe covering the client's arms and body, with a shawl covering the head and neck. Based on this information, which supplement will the nurse most likely anticipate providing to the client? A. Vitamin D B. Vitamin C C. Calcium D. Zinc

Explanation Choice A is correct. Female clients of Middle Eastern descent who continue to follow Middle Eastern traditions will often dress in traditional Middle Eastern attire. Although this attire varies based on the specific region from which the client's descents originated, as well as any specific religious beliefs held, traditions generally dictate that women are covered from head to toe. Dressing in this manner results in reduced sun exposure on the client's skin. Unless the client's diet is rich in good sources of vitamin D, the client will need vitamin D supplementation. Vitamin D deficiency in a pregnant woman causes deficiency in the fetus. Choice B is incorrect. There is no correlation between this situation and the need for vitamin C supplementation. Choice C is incorrect. There is no direct correlation between this situation and the need for calcium supplementation unless the vitamin D level is at such a diminished level that the calcium level has subsequently become negatively impacted (e.g., hypocalcemia). Choice D is incorrect. There is no correlation between this situation and the need for zinc supplementation. Learning Objective When caring for a client of Middle Eastern descent who is currently 24 weeks gestation and who wears the traditional head-to-toe cultural garments, recognize that vitamin D is the supplement that the nurse should most likely anticipate providing the client. Additional Info Vitamin D deficiency may result from inadequate exposure to sunlight. Occasionally, in pregnant women, vitamin D deficiency severe enough to cause maternal osteomalacia results in rickets with metaphyseal lesions in neonates. In young infants, rickets causes softening of the entire skull (craniotabes). When palpated, these clients' occiput and posterior parietal bones may indent easily.

The nurse understands that which of the following are complications of acute tubular necrosis (ATN)? Select all that apply. A. Metabolic acidosis B. High thyroxine levels C. Hyponatremia D. Decreased parathyroid levels E. Electrolyte imbalances

Explanation Choice A is correct. The kidneys cannot excrete excess hydrogen ions or reabsorb bicarbonate with ATN. Due to the inability to excrete the excess acid (hydrogen ions) paired with the inability to hang on to the needed base (bicarbonate), acidosis ensues. This is due to the malfunction of the kidneys, not the lungs, so it is classified as metabolic acidosis. Choice C is correct. ATN can cause hyponatremia. Due to lower urinary output, there is hypervolemia. With fluid retention and high volume remaining in the blood vessels, the amount of sodium in the body is diluted. This is called relative dilutional hyponatremia. Choice E is correct. The kidneys play a crucial role in regulating the levels of electrolytes such as sodium, potassium, and calcium in the body. ATN can disrupt this process, leading to imbalances that can cause a range of symptoms, including muscle weakness, confusion, and heart rhythm abnormalities. Choice B is incorrect. ATN is associated with low thyroxine levels, not high. Thyroid hormones increase renal blood flow and glomerular filtration rate (GFR). In ATN, there is often lower renal blood flow and a lower GR. Therefore, ATN is often associated with low thyroid levels. Choice D is incorrect. ATN can cause increased parathyroid levels. This is considered a secondary hyperparathyroidism. Secondary hyperparathyroidism occurs when the parathyroid glands release too much parathyroid hormone (PTH), causing a high blood level of PTH. This occurs in ATN because when the kidneys are damaged, they cannot make active vitamin D. Vitamin D is required for the absorption of calcium, and calcium levels are therefore low in patients with ATN. One of the primary functions of PTH is the release of calcium from the bones, into the bloodstream, when blood calcium levels are low. The body recognizes the lower blood calcium level, that has been caused by the ATN, and then secrects more PTH to try to correct the issue. This is when ATN can cause increase PTH levels. Additional Info ✓Acute tubular necrosis (ATN) is a medical condition that occurs when there is damage to the tubular cells of the kidneys. ✓The tubules are responsible for filtering waste and excess fluids from the blood and excreting t

Chemotherapy induces vomiting by: A. Stimulating neuroreceptors in the medulla. B. Inhibiting the release of catecholamines. C. Autonomic instability. D. Irritating the gastric mucosa.

Explanation Choice A is correct. Vomiting (emesis) is initiated by a nucleus of cells located in the medulla called the vomiting center. This center coordinates a complex series of events involving pharyngeal, gastrointestinal, and abdominal wall contractions that lead to the expulsion of gastric contents. Choice B is incorrect. Catecholamine inhibition does not induce vomiting. Choice C is incorrect. Chemotherapy does not induce vomiting from autonomic instability. Choice D is incorrect. Chemotherapy, especially oral agents, may have an irritating effect on the gastric mucosa, which could result in afferent messages to the solitary tract nucleus. Still, these pathways do not project to the vomiting center. NCSBN Client Need Topic: Physiological Integrity, Subtopic: Pharmacological Therapies, Drugs for Neoplasia

The nurse plans care for a client with moderate Alzheimer's disease (AD). Which of the following interventions should the nurse include? A. Provide a low-stimulation environment with adequate lighting B. Quiz the client with orientation questions C. Change assigned staff to avoid burnout D. Provide a broad range of choices

Explanation Choice A is correct. When caring for a client with moderate Alzheimer's disease, the nurse should provide a low-stimulated environment with adequate lighting. The rationale for giving a low-stimulated environment is to allow the client to cognitively process one task at a time and not become distracted. Choice B is incorrect. Quizzing the client would result in frustration. The care plan should cue the client with necessary cues and avoid confounding the client. Choice C is incorrect. Consistent caregivers are always a good idea because it enriches the client's relationship with the caregiver and provides comfort from familiarity. Choice D is incorrect. Providing a broad range of choices is not recommended because this may lead to indecisiveness, causing the client to feel frustrated. Additional Info ✓ Alzheimer's disease (AD) is the most common type of dementia ✓ The progressive deterioration of cognitive functioning commonly characterizes AD. Initial deterioration may be so subtle and insidious that others may not notice it ✓ In the early stages of the disease, the affected person may be able to compensate for and hide cognitive deficits ✓ Manifestations of AD include impairment of recent memory which progresses to remote memory ✓ As the disease progresses, the client may have impairment in executive functioning, including planning, organizing, and problem-solving ✓ Additional clinical features of AD include the triple-A (agraphia, agnosia, and apraxia) ✓ Nursing care is aimed at maximizing function and promoting safety ✓ The nurse should also provide caregiver/family support because of the emotional toll of this disorder

The nurse is discussing ocular disorders with a group of nursing students. Which of the following statements would be correct for the nurse to make? Select all that apply. Cataracts are caused by increased ocular pressure (IOP). Graves' disease may cause exophthalmos. Macular degeneration is manifested by loss of peripheral vision. Angle-closure glaucoma is manifested by headache and eye pain. Hyphema results in increased aqueous humor in the anterior chamber.

Explanation Choice B and D are correct. Graves' disease may cause a client to develop exophthalmos. Angle-closure glaucoma is a medical emergency where the IOP is greater than 30 mmHg, and the client has manifestations such as eye pain, headache, blurred vision, and reddened eye appearance. Choices A, C, and E are incorrect. Increased IOP is a central feature of glaucoma. Cataracts is a disorder of the lens as it causes the client to have difficulty discriminating colors and seeing in low light. Opacities can commonly be seen in the affected eye. Macular degeneration causes central vision loss, not vision loss in the peripheral fields. A hyphema is caused by blood in the eye's anterior chamber. NCLEX Category: Physiological Adaptation Activity Statement: Illness management Question type: Analysis Additional Info In managing a hyphema and angle-closure glaucoma, the nurse should be aware of the following: A hyphema is an ocular emergency that has been caused by blood in the anterior chamber. This injury results from trauma and should be addressed promptly with interventions such as elevating the head of the bed to 30 degrees and shielding the affected eye. Angle-closure glaucoma is an ocular emergency that requires the client to receive prescribed agents such as timolol to lower intraocular pressure. The client should be placed supine, which will assist in the lens falling away from the iris, decreasing the pupillary block.

The nurse is caring for a client with akathisia. The nurse should anticipate a prescription for which medication? Omitted A. Modafinil B. Propranolol C. Venlafaxine D. Duloxetine

Explanation Choice B is correct. Akathisia is the most common extrapyramidal side effect (EPS) associated with antipsychotic medications. Propranolol is an effective treatment for akathisia as this helps with treating the internal sense of restlessness characterized by this effect. Choices A, C, and D are incorrect. Modafinil is a psychostimulant used in the treatment of narcolepsy. This would likely make akathisia worse and would not be indicated. Venlafaxine and duloxetine are serotonin-norepinephrine reuptake inhibitors (SNRIs) and are not used to manage akathisia. Additional Info Akathisia is a sense of motor restlessness and is one of the most common EPS. The individual feels a compelling urge to move and could be mistaken as an individual being aggressive or agitated. Medications that can cause EPS include: antipsychotics (haloperidol, fluphenazine) and other dopamine-modulating medications such as metoclopramide. Prior to administering propranolol, the nurse must obtain the client's blood pressure and pulse. Hypotension and bradycardia would be contraindicated to administering this medication.

The nurse is performing an admission assessment on a client admitted to the behavioral health unit. The client is reporting new-onset blindness after witnessing a traumatic motor vehicle accident. The nurse suspects that this client is using which defense mechanism? A. suppression B. conversion C. displacement D. dissociation

Explanation Choice B is correct. Converting anxiety into physical symptoms with no organic cause best explains this defense mechanism this client is experiencing. Conversion is a pathological defense that may manifest as a disorder if it continues to recur. The accident traumatized this client and converted his anxiety into a physical symptom (blindness). His new-onset blindness has no organic origin; thus, this exemplifies conversion. Choices A, C, and D are incorrect. Suppression is defined as the conscious decision to delay addressing a disturbing situation. The client does not exhibit this avoidance because they have taken their anxiety and manifested it as a physical ailment that cannot be explained. Displacement is the transference of emotions associated with a particular person, object, or situation to another non-threatening person, object, or situation. This client has not transferred their anxiety to someone (or something). Finally, dissociation is a disruption in consciousness, memory, identity, or perception of the environment that results in compartmentalizing uncomfortable or unpleasant aspects of oneself. This client has no evidence of a disruption in their consciousness, memory, or identity. Additional Info ✓ Adaptive use of defense mechanisms helps people lower their anxiety levels and achieve their goals in acceptable ways. ✓ Maladaptive use of defense mechanisms occurs when one or several are used to excess, particularly immature defenses. ✓ Most defense mechanisms can be used in both healthy and unhealthy ways. People generally use a variety of defense mechanisms but not always to the same degree. ✓ Defenses range from immature (e.g., projection, denial, and splitting) to mature (e.g., altruism, sublimation, and humor).

The nurse is caring for a client with narcolepsy. Which of the following medications would the nurse anticipate the primary healthcare provider (PHCP) prescribe? A. Aripiprazole B. Modafinil C. Ropinirole D. Quetiapine

Explanation Choice B is correct. Modafinil is a psychostimulant that is effective in treating narcolepsy. This medication promotes wakefulness and is dosed either once or twice a day. Choices A, C, and D are incorrect. Aripiprazole and quetiapine are atypical antipsychotics and not indicated in the management of narcolepsy. Quetiapine is highly sedating and would be counterproductive in the management of narcolepsy. Ropinirole is a dopaminergic and is indicated in treating Parkinson's disease and restless leg syndrome. NCLEX Category: Pharmacological and Parenteral Therapies Activity Statement: Expected actions/outcomes Question type: Knowledge/comprehension Additional Info Narcolepsy is a syndrome in which a client has significant daytime sleepiness that often lessens after a nap. Stimulants such as modafinil and armodafinil are indicated in the management of narcolepsy as they promote wakefulness. Common side effects of modafinil include headache, nervousness, anorexia, and weight loss.

The nurse is assessing a client who has suspected Raynaud phenomenon/disease. Which of the following findings would support a diagnosis of Raynaud phenomenon/disease? A. unilateral swelling of the leg B. painful vasospasms C. crepitus of the joints D. claudication in feet and lower extremities

Explanation Choice B is correct. Raynaud phenomena is a condition causing painful vasospasms in response to emotional stress and cold temperatures. These painful vasospasms occur in the digits. Raynaud phenomena may be associated with autoimmune conditions such as systemic lupus erythematosus (SLE) and scleroderma. Choice A is incorrect. Unilateral swelling of the leg may be linked to venous thromboembolism. This is not a clinical feature of Raynaud phenomena. Choice C is incorrect. Crepitus of the joints would be a clinical feature associated with osteoarthritis, not Raynaud phenomena. Choice D is incorrect. Claudication of the lower extremities is pain experienced as a client ambulates and is relieved with rest. This is a classic feature of peripheral arterial disease, not Raynaud phenomena. Additional Info ✓ Raynaud disease/phenomena is a condition that causes painful vasospasms in the digits when the client experiences stress or is exposed to cold temperatures ✓ This condition may cooccur with autoimmune conditions such as lupus or scleroderma ✓ Nursing care involves administering prescribed calcium channel blockers, instructing clients to wear gloves if they come into contact with cold surfaces, and protecting their fingers from injury.

The nurse is caring for a client in her second trimester who presents to the maternity clinic expressing concern that the dark, vertical line present on the midline of her abdomen may pose a danger to her baby. Which of the following would be the most appropriate action for the nurse to take in response to the client's concern? A. Refer the client to a dermatologist for assessment. B. Educate the client that this is a common occurrence in pregnancy called linea nigra, which usually disappears after childbirth. C. Ask the client what types of foods she has been ingesting. D. Educate the client that this is a common occurrence in pregnancy called linea nigra, which typically remains following childbirth.

Explanation Choice B is correct. The linea alba is the line that marks the longitudinal division of the midline of the abdomen—darkens to become the linea nigra due to hormone changes during pregnancy. This dark pigmentation may extend from the symphysis pubis to the top of the fundus and becomes darker as pregnancy progresses. This hyperpigmentation is temporary and typically disappears after childbirth due to hormonal changes during pregnancy. Choice A is incorrect. There is no need for further assessment or evaluation by a dermatologist, as this is a common occurrence of pregnancy. Choice C is incorrect. There is no connection between food intake and the appearance of the linea nigra. Choice D is incorrect. Hormonal changes during pregnancy cause temporary pigmentation darkening making the linea nigra more visually prominent in some women. This hyperpigmentation is not permanent and will typically disappear following childbirth. Learning Objective Recognize that a linea nigra is a temporary hyperpigmentation that appears vertically down the abdomen due to hormone changes during pregnancy. Additional Info ✓ Those with darker complexions tend to have a more pronounced linea nigra than those with fair complexions. ✓ In most individuals, the linea nigra darkens enough to be visible in the second trimester. ✓ Following delivery, fading is gradual, occurring over several weeks or months. ✓ The exact cause of linea nigra is unknown, but the hypothesis is that the melanocyte-stimulating hormone created by the placenta causes melanin to rise during pregnancy

The nurse is educating a patient who is taking phenytoin. To make sure phenytoin does not fail, which over-the-counter (OTC) medication should the nurse advise the patient not to take at the same time? A. Acetaminophen B. Ibuprofen C. Calcium carbonate D. Ranitidine

Explanation Choice C is correct. Calcium carbonate (Tums) should not be taken at the same time as Phenytoin because taking them together can decrease the effects of phenytoin. Antacids containing calcium carbonate reduce the bioavailability of phenytoin by reducing both the rate of absorption and the amount of intake. Phenytoin is an anticonvulsant and not getting it at a therapeutic dose may result in the client having a recurrent seizure. Clients should be cautioned against the concomitant use of antacids/tums and phenytoin. If the client needs calcium carbonate, he should be instructed to separate the times of intake of calcium carbonate and phenytoin by at least two to three hours. Choice A is incorrect. Acetaminophen and phenytoin can be taken together without any concern for therapeutic failure. Choice B is incorrect. Ibuprofen and phenytoin can be taken together and do not cause the therapeutic failure of phenytoin. Choice D is incorrect. Ranitidine and phenytoin can be taken together and do not cause the therapeutic failure of phenytoin. Ranitidine may, however, increase the effects of Phenytoin, so the patient should be monitored for any phenytoin-related adverse effects.

A client in a psychiatric clinic tells the nurse, "I want to kill my wife. The moment I see her, I am going to kill her." Which of the following should be the nurse's next action? A. Respect the client's right to privacy and confidentiality B. Document the client's statements C. Notify the client's psychiatrist of the comments D. Explore the client's feelings about his wife

Explanation Choice C is correct. Confidentiality plays a critical role in client care; however, there may be certain circumstances where confidentiality must be breached to not only ensure the safety of the client, but also to protect a third party (or parties). This concept is referred to as the 'duty to warn' or 'duty to protect.' These types of situations most often arise when a client reports suicidal ideation (SI), homicidal ideation (HI), or when the client makes a threat against an identifiable third party, even if the threat was made during a private therapy session. Choice A is incorrect. As mentioned above, confidentiality plays a critical role in client care; however, there may be certain circumstances where confidentiality must be breached to not only ensure the safety of the client, but also to protect a third party (or parties). Choice B is incorrect. Clear and accurate documentation of the client's statements and the context of the conversation is vital; however, the priority for the nurse is to ensure the safety of the client and the client's wife, likely best achieved by prompt reporting of the client's statements to the client's psychiatric health care provider (HCP). Choice D is incorrect. Assisting the client in exploring their feelings regarding their wife would potentially further increase the client's anger toward her. Therefore, this is not an appropriate action for the nurse to take at this time. Learning Objective When providing care for a client who threatens to kill a third party, recognize that notifying the client's psychiatric health care provider (HCP) of the client's comments is the next action for the nurse to take. Additional Info There is a wide range of 'duty to warn' and 'duty to protect' legal variations across the country, with nurses being impacted in varying degrees. It is crucial for all health care providers (HCPs) of all levels to know the laws in the state(s) in which they practice, as there are significant legal variations from state to state. Failure to know the applicable state statutes and/or regulations may lead to inadvertent violations of the Health Insurance Portability and Accountability Act of 1996.

The nurse is educating a client about newly prescribed aspart insulin. The nurse should instruct the client to self-administer this insulin A. 30-45 minutes before a meal B. one hour after a meal C. 20-30 minutes before a meal D. 5-10 minutes before a meal

Explanation Choice D is correct. Aspart insulin is a rapid-acting insulin that should be administered to the client no greater than 5-10 minutes prior to the meal or while the client is actively eating. Before administering this insulin, the client's blood glucose should be obtained. Choices A, B, and C are incorrect. These are inappropriate times to administer aspart insulin. Rapid onset insulins (lispro, aspart, glulisine) are given 5-10 minutes before a meal or while the client is actively eating. Additional Info ✓ The three rapid-acting insulins are lispro, aspart, and glulisine. ✓ The client needs to take this insulin 5-10 minutes before a meal or while actively eating. ✓ A rapid-acting insulin is utilized as correctional insulin before meals to prevent post-prandial hyperglycemia. ✓ This type of insulin is commonly loaded into an insulin pump.

The nurse is caring for a client who has had an exacerbation of Bell's palsy. The client is experiencing paralysis of their eye, the nurse should plan to A. tape an eye patch to the affected eyelid at all times. B. instruct the client to keep both eyes closed. C. assess the pupil's size and reactivity to light. D. apply the prescribed ocular lubricant to the affected eye.

Explanation Choice D is correct. Bell's palsy is a lower motor neuron facial nerve palsy that can result in the weakness of facial muscles and the muscles responsible for eye closure (orbicularis oculi). A client with Bell's palsy who cannot blink would be unable to close the affected eye. As a result, the cornea becomes overly dry, leading to an increased risk of corneal ulceration and scarring. Eye lubricant (i.e., typically artificial tears) must be applied as often as every hour during the day to keep the eye moist and prevent corneal drying. A moisturizing eye ointment may be used at night. Choices A, B, and C are incorrect. Applying an eye patch with tape on the eyelid may cause the patch to slip into the open eye and cause a corneal abrasion. During the day, the client should protect the open eye with glasses or goggles. At night, the client may use a soft eye patch to cover the open eye, but it should not be taped to the eyelid. Instead, the soft eye pad should be secured with one end of the tape on the client's forehead and the other end on the cheek diagonally. It is not necessary for the client to keep the unaffected eye closed. Bell's palsy does not affect the pupil's reaction to light and accommodation. Learning Objective Recognize that when caring for a client with Bell's palsy, the plan of care should include an order from the health care provider (HCP) to apply eye lubricant to prevent corneal drying.

The nurse is caring for a child immediately following a nephrectomy in the postanesthesia care unit (PACU). Which assessment should the nurse initially perform? A. pain level B. peripheral vascular access device C. surgical incision D. vital signs

Explanation Choice D is correct. Immediately following surgery, the nurse needs to determine the client's physical stability by obtaining and assessing vital signs. General anesthesia may cause a client to experience respiratory depression, and the only way this may be determined is through the vital signs (respiratory rate and pulse oximetry). The client's pulse will also need to be ascertained because if it is tachycardia, it may indicate shock. Choice A is incorrect. Assessing the client's pain level is part of the postoperative assessment, but it would not be as valuable as the client's vital signs. The client's pain level would not yield any information if the client would be experiencing respiratory depression, shock, or hypothermia. Choice B is incorrect. Assessing the PVAD is part of a nursing assessment but would not prioritize the client's vital signs, which are clinically significant within the immediate postoperative period. Choice C is incorrect. Assessing the surgical incision is not a priority compared to determining the client's overall physical status. Vital signs during the immediate postoperative period are highly valuable because they may indicate if the client is experiencing hypothermia, respiratory distress, or shock. Additional Info ✓ Immediate postoperative assessments include - Vital signs Surgical incision Pain level Peripheral vascular access device Nausea and vomiting Urine output Postoperative laboratory data (if ordered), including complete blood count and basic metabolic panel

The nurse is caring for a client who recently had a total parathyroidectomy. Which of the following medications should the nurse anticipate that the primary health care provider (PHCP) will order? Select all that apply A. Calcium carbonate B. Cholecalciferol C. Calcitonin D. Folic acid E. Magnesium oxide

Explanation Choices A and B are correct. Following a parathyroidectomy, aggressive calcium replacement typically commences. Two medications commonly prescribed include cholecalciferol (Vitamin D3) and calcium carbonate. Cholecalciferol is necessary to enhance the absorption of calcium carbonate. Calcium levels are monitored closely following this procedure. The parathyroid regulates calcium via the release of parathyroid hormone. Choices C, D, and E are incorrect. Calcitonin would be contraindicated because this medication would lower serum calcium levels. Folic acid and magnesium oxide replacement are irrelevant to the parathyroid. Magnesium oxide would be beneficial for an individual with alcoholism.

Which of the following are appropriate nursing interventions to prevent aspiration after a child has vomited? Select all that apply. Position the child on their side. Suction the mouth to remove vomitus. Offer the child a sip of water to clear the mouth. Assess the character and amount of vomitus. Ask primary healthcare provider about antiemetic medication

Explanation Choices A and B are correct. Positioning the child on their side will prevent aspiration and maintain a patent airway (Choice A). Suctioning the mouth will remove any further vomitus keeping the mouth clean and preventing aspiration (Choice B). Choice C is incorrect. It is not safe to offer the child a sip of water at this time as they may aspirate on thin liquids. The child first needs to be assessed and the problem identified before it is decided that they are safe for oral intake. Chocie D is incorrect. Although it is an appropriate nursing intervention to assess the character and amount of emesis, this does not do anything to prevent aspiration after the child has vomited. Choice E is incorrect. A child that is vomiting may benefit from an antiemetic, but this does not prevent aspiration in the child who has already vomited. NCSBN Client Need: Topic: Physiological Integrity, Subtopic: Reduction of risk potential, Gastrointestinal disorders Additional Info Prevention of aspiration in infants and young children include: ✓ feeding in upright or prone position ✓ avoid feedings directly before laying flat for sleep Antiemetic medication therapy for pediatrics is ondansetron, which is a serotonin receptor antagonist.

The nurse is caring for a client with a sodium level of 130 mEq/L(135-145 mEq/L). Which of the following medications may cause this abnormality? Select all that apply. Spironolactone Hydrochlorothiazide Prednisone Sodium polystyrene Tolvaptan

Explanation Choices A and B are correct. Spironolactone is a diuretic that retains potassium but causes the loss of water and sodium. Hydrochlorothiazide is a thiazide diuretic that may contribute to hyponatremia because while it does raise serum calcium levels, it depletes every other electrolyte. Choices C, D, and E are incorrect. Prednisone is a corticosteroid used for inflammatory conditions. This drug causes an increase in aldosterone, which increases sodium and water retention. Sodium polystyrene is used for individuals with hyperkalemia, and its use will not only lower potassium but may also raise sodium. Tolvaptan is a medication used to treat syndrome of inappropriate antidiuretic hormone (SIADH). It depletes the water but not the sodium. Additional Info ✓ Hyponatremia is sodium less than 135 mEq/L. ✓ The cause of hyponatremia is multifactorial and may include diuretics, lithium, alcoholism, and certain forms of dehydration. ✓ For severe hyponatremia, the nurse should institute seizure precautions.

The nurse is assessing a child with intussusception. Which of the following findings would be expected? Select all that apply. Red, currant jelly stool Hematemesis Palpable, sausage-shaped abdominal mass Steatorrhea Jaundice

Explanation Choices A and C are correct. Red, currant jelly stool is a classic finding of intussusception. When the bowel telescopes into another portion of the intestine, it causes intestinal obstruction and rectal bleeding - subsequently, red, currant jelly stools (choice A). A palpable, sausage-shaped mass in the right lower quadrant (RLQ) is another classic finding of intussusception. This is due to the physical telescoping of the intestine, and the mass can sometimes be felt on palpation (choice C). Choices B, D, and E are incorrect. Hematemesis, or bloody vomiting, is not an expected finding in intussusception. Vomiting of green bile (bilious) is seen due to obstruction associated with intussusception (choice B). Steatorrhea is the passage of oily, pale, foul-smelling stool. Steatorrhea indicates fat malabsorption and can be a sign of Celiac disease, but it would not be present in a client with intussusception (choice D). Jaundice is a clinical feature seen with hepatitis infection or other liver diseases, but not intussusception (choice E). Additional Info ✓ Intussusception is a bowel obstruction commonly occurring between children between 3 and 6 years old ✓ A proximal segment of the bowel telescopes into a more distal segment, pulling the mesentery with it, causing compression ✓ Manifestations of intussusception include abrupt abdomen pain, vomiting, lethargy, the passage of red, currant jelly-like stools (stool mixed with blood and mucus), abdominal distention, and palpable sausage-shaped mass in the abdomen. ✓ Passage of a normal brown stool usually indicates that the intussusception has reduced itself ✓ Nursing care includes hydrating the child, administering prescribed antibiotics and pain medications, and potentially preparing the child for surgery

The nurse is caring for a client who was just intubated via an endotracheal tube (ETT). The nurse anticipates that the placement of the tube will be verified by Select all that apply. chest x-ray. cuff pressure. chest wall movement. end-tidal carbon dioxide (EtCO2). arterial blood gas (ABG).

Explanation Choices A and D are correct. When a client is intubated via ETT, the initial verification methods include a chest x-ray and end-tidal carbon dioxide (EtCO2). A chest x-ray will verify that the tube terminates 2-3 cm above the carina. End-tidal carbon dioxide (EtCO2) monitoring is a device that may be added and typically changes color when the tube is in the appropriate place. End-tidal carbon dioxide ranges between 20 and 40 mm Hg. Choices B, C, and E are incorrect. The cuff pressure is a parameter that is monitored to ensure that the ETT is appropriately anchored and allows for effective gas exchange through the tube and to deliver the tidal volume if the client were to receive mechanical ventilation. Normal pressure should range between 14-20 mmHg. Chest wall movement is not a definitive way to discern that the ETT is in the correct place. While this is a key assessment following intubation, it would not discern the definitive placement of the ETT. ABGs are frequently done during the course of a client being mechanically ventilated. However, they do not inform the nurse of the tube placement. Additional Info ✓ When a client is being mechanically ventilated, airway patency is the priority. ✓ A bag-valve mask should be readily available during a power failure or emergency resuscitation. ✓ Review ventilator settings frequently and collaborate with respiratory therapy to assist in ventilator management. ✓ The client's head of the bed should be more than 30 degrees when supine to decrease the risk for aspiration and ventilator-associated pneumonia (VAP). ✓ VAP can be prevented by Oral care using an antiseptic Minimizing exposure to proton pump inhibitors Meticulous hand hygiene by nursing staff/visitors Pulmonary hygiene measures such as chest physiotherapy When caring for a client on a ventilator, you should be familiar with the following settings: Mode (Volume [SIMV, A/C] or Pressure [PSV]) Rate (Number of breaths per minute) Tidal volume (the amount of gas delivered to the client) Fraction of inspired oxygen (FiO2 - the percentage of oxygen given per breath) PEEP (pressure added at exhalation to keep the small airways open and mitigate atelectasis) Pressure support (PS - provides added pressure w

The case manager is reviewing the medical record of a client with schizophrenia Item 1 of 1 Progress Notes Discharge Summary 0900: Third involuntary admission in the past six months. The client was admitted four days ago because of florid psychosis. During the stay, the client was stabilized with their prescribed ariprazole. Once stabilized, the client reported nonadherence to aripiprazole because they 'forget.' It is documented that the client also missed two follow-up appointments for some unknown reason. The client was prescribed aripiprazole oral distinguishing tablets (ODT) to optimize adherence versus tablets. Provided two refills and a follow-up appointment at discharge. Thorough counseling was provided regarding the dosing schedule. Considering the client's repeated nonadherence, there is a high probability of readmission. Will consult case management for follow-up Orders 0815: discharge client home case management consultation for repeated readmissions arrange outpatient follow-up appointment prior to discharge give morning dose of aripiprazole prior to discharge discharge with a prescription for aripiprazole 15 mg ODT daily The case manager reviews the physician's progress n

Explanation Choices A and E are correct. Epilepsy is an idiopathic condition that requires maintenance treatment by using anticonvulsants. Topiramate is an anticonvulsant that may be used in the prevention of seizures. Lorazepam is also indicated in epilepsy in the event of a patient experiencing an acute seizure. The topiramate should be used for maintenance purposes, and the lorazepam would be indicated for an acute seizure. Choices B, C, and D are incorrect. Risperidone is indicated for psychotic disorders such as schizophrenia. Prazosin is an antihypertensive that may be used for high blood pressure. This medication also may be indicated for psychiatric illnesses such as PTSD. Hydroxyzine is indicated for anxiety disorders as well as allergic rhinitis. Additional Info Source : Archer Review Epilepsy is an idiopathic condition that requires management with anticonvulsants such as topiramate, valproic acid, or phenytoin. Acute seizures are managed with benzodiazepines such as lorazepam or diazepam. These medications work to terminate a seizure. During an acute seizure, the nurse should place the patient on their side, loosen restrictive clothing, and anticipate a prescription for a parenteral benzodiazepine such as diazepam.

The nurse is caring for a client diagnosed with epilepsy. The nurse should anticipate a prescription for which of the following medications? Select all that apply. Topiramate Risperidone Prazosin Hydroxyzine Lorazepam

Explanation Choices A and E are correct. Epilepsy is an idiopathic condition that requires maintenance treatment by using anticonvulsants. Topiramate is an anticonvulsant that may be used in the prevention of seizures. Lorazepam is also indicated in epilepsy in the event of a patient experiencing an acute seizure. The topiramate should be used for maintenance purposes, and the lorazepam would be indicated for an acute seizure. Choices B, C, and D are incorrect. Risperidone is indicated for psychotic disorders such as schizophrenia. Prazosin is an antihypertensive that may be used for high blood pressure. This medication also may be indicated for psychiatric illnesses such as PTSD. Hydroxyzine is indicated for anxiety disorders as well as allergic rhinitis. Additional Info Source : Archer Review Epilepsy is an idiopathic condition that requires management with anticonvulsants such as topiramate, valproic acid, or phenytoin. Acute seizures are managed with benzodiazepines such as lorazepam or diazepam. These medications work to terminate a seizure. During an acute seizure, the nurse should place the patient on their side, loosen restrictive clothing, and anticipate a prescription for a parenteral benzodiazepine such as diazepam.

A 12-year-old is evaluated for generalized pain in the emergency department (ED) Item 1 of 1 History and Physical A 12-year-old male with sickle cell anemia presents with his father with generalized pain that worsened overnight. The client stated his pain is in his knees and ankles. His knees and ankles had a full range of motion on exam and were tender to the touch. Slight swelling in the ankles and knees was observed. The client states that the pain 'hurts a lot and he had difficulty sleeping with the pain. He rates his pain as a '7' on a scale from 0-10. The father denies any recent illnesses or any strenuous activity. The child's vaccines are current, and he takes no medication daily. Orders Establish peripheral vascular access Start patient-controlled analgesia, intravenous (IV) morphine 250 mL of 0.9% saline over one-hour Complete blood count (CBC); Complete metabolic panel (CMP) Vital Signs Temperature 98.0o F (37o C) Pulse 93/minute Respirations 16/minute Blood Pressure 113/81 mm Hg Oxygen saturation 96% on room air The nurse is implementing care for this client Click to specify if each nursing action is appropriate or not appropriate Nursing Action Appropriate, N

Explanation Regardless of the prescribed PCA setting, the nurse should verify the settings with a second nurse as the pump is being initiated. This helps reduce errors with morphine which is a high-risk medication. These settings should also be verified during the handoff report and when prescribed changes are made. It would be appropriate for the nurse to apply EMLA (topical lidocaine) via a disk or thick cream to the skin before starting an IV to decrease the discomfort associated with the procedure. This cream is applied 30 to 60 minutes prior to starting the IV. It would not be appropriate for the nurse to administer oxygen via nasal cannula as this client has an optimal oxygen saturation. It is detrimental to administer supplemental oxygen to a client with SCA who is appropriately oxygenating. Oxygen does not reverse sickled RBCs; it decreases erythropoiesis if used in a nonhypoxic patient. It is not necessary to keep the client NPO. It would be beneficial to encourage the client to increase their intake of fluids and fluid-rich foods such as fruit. Finally, warm compresses effectively provide pain relief because of the vasodilation effect. Additional Info For a child with a vasoocclusive crisis, the nurse must work to mitigate pain and discomfort with prompt initiation of peripheral vascular access. The PCA device is an effective mechanism to deliver pain control as well as promote client autonomy. PCAs may be utilized to deliver a preset amount of pain medication in an on-demand dose or continuously as a basal dose. Additional treatments for a vasoocclusive crisis include intravenous fluids, rest, and warm compresses to the joints.

The nurse is caring for a client diagnosed with a myxedema coma. The nurse should anticipate a prescription for which of the following medications? Select all that apply. Levothyroxine Methimazole Tolvaptan Hydrochlorothiazide Hydrocortisone

Explanation Choices A and E are correct. When a client experiences a myxedema coma, it is because of severe hypothyroidism. These dangerously low levels of thyroid hormone produce symptoms such as altered level of consciousness, hyponatremia, hypothermia, hypoventilation, and hypoglycemia. Treatment is essential and is geared towards the prompt administration of intravenous levothyroxine and liothyronine. Glucocorticoids are usually added to the treatment to help mitigate the hypotension and potential overlook of adrenal dysfunction. Choices B, C, and D are incorrect. Methimazole would be contraindicated in myxedema since this is a type of antithyroid medication. Furthermore, tolvaptan is not indicated because this medication is used to treat SIADH. HCTZ is a treatment for essential hypertension and nephrogenic diabetes inspidus. It has no role in a myxedema coma. Additional Info ✓ Myxedema coma is a rare but extremely serious complication associated with severe hypothyroidism. ✓ Manifestations of a myxedema coma include hyponatremia, hypothermia, hypoventilation, and hypoglycemia. ✓ The nurse must initiate medical and symptomatic treatment, such as intravenous levothyroxine and hydrocortisone. ✓ The nurse may also treat the client's symptoms with passive rewarming.

The nurse is working with a client who has been diagnosed with hypervolemia. Which of the following conditions can cause hypervolemia? Select all that apply. Heart failure Renal failure Type 1 Diabetes Mellitus Third degree burns Hormonal imbalances

Explanation Choices A, B and E are correct. Heart failure can cause hypervolemia. When the heart is not pumping effectively, there is decreased cardiac output. This means less perfusion to all of the body's organs, including the kidneys. When the kidneys don't get enough blood, the urinary output will decrease; instead of the body getting rid of fluid in the urine, the volume will stay in circulation and cause hypervolemia (Choice A). Renal failure can cause hypervolemia. If the kidneys are failing, they are not effectively making urine. If the body is not excreting fluid in the urine, that fluid is staying in the vascular space and causes hypervolemia (Choice B). Hormonal imbalances, such as those caused by excessive production of cortisol or aldosterone, can lead to an increase in fluid retention and hypervolemia (Choice E). Choices C and D are incorrect. Type 1 diabetes mellitus (Choice C) would not cause hypervolemia. In a well-controlled diabetic, there should be no fluid imbalance. However, if the diabetic is not well controlled and is hyperglycemic, they will be polyuric. This is because the increased amount of glucose in the vascular space will induce diuresis. Polyuria leads to decreased volume in the vascular area, hypovolemia, the opposite of hypervolemia. Third-degree burns can cause hypovolemia (Choice D), not hypervolemia. This is due to the large amount of fluid that leaves the vascular space and goes out into the interstitial space after a burn. NCSBN Client Need: Topic: Physiological Integrity, Subtopic: Physiological adaptation, Fluid & Electrolytes Additional Info Some contributing factors in the development of hypervolemia can be the following: ✓Heart failure: When the heart is unable to pump blood effectively, fluid can build up in the lungs and other parts of the body. ✓Kidney disease: The kidneys are responsible for removing excess fluid from the body. When they are not functioning properly, fluid can accumulate. ✓Liver disease: Liver disease can lead to a decrease in albumin production, which can result in fluid accumulation. ✓Excessive fluid intake: Receiving excessive IV fluids can lead to hypervolemia. ✓Hormonal imbalances: Hormones such as aldosterone can affect fluid balance in t

The nurse is developing a staff in-service on negligence. It would indicate correct understanding if the participant states that which element must be met in a negligent lawsuit? Select all that apply. Duty owed Breach of duty owed Causation Harm or damages Beneficence

Explanation Choices A, B, C, and D are correct. For a negligent lawsuit to proceed, the plaintiff (injured client) must prove the following elements: 1. A duty of care was owed to the injured party. 2. There was a breach of that duty. 3. The breach of the duty caused the injury (causation). 4. The plaintiff suffered actual harm or damages. Choice E is incorrect. Beneficence is an ethical principle and has no legal implications in negligence. Beneficence is defined as an individual acting in positive regard for others with a kind spirit. Additional Info Negligence is defined as the failure to exercise the care toward others that a reasonable or prudent person would do in the circumstances or taking action that such a reasonable person would not. The plaintiff must establish all four elements if a negligent lawsuit can be litigated.

The nurse is assessing a child with iron deficiency anemia. Which of the following would be an expected finding? Select all that apply. Tachycardia Pica Pallor Insomnia Fatigue

Explanation Choices A, B, C, and E are correct. Tachycardia is an expected assessment finding for a client with iron deficiency anemia. When the client has decreased oxygen delivery to the tissues, the body increases the heart rate to compensate. Pica is an expected assessment finding for a client with iron deficiency anemia. Pica is defined as "a tendency or craving to eat substances other than normal food (such as clay, plaster, or ashes)" This is due to the low iron level in the body. Pallor is an expected assessment finding for a client with iron deficiency anemia. Due to low iron levels, there is decreased oxygen delivery to the tissues and reduced perfusion. This causes pallor and other signs of decreased perfusion. Fatigue is a non-specific finding in anemia because of the decreased circulating hemoglobin that supplies oxygen to organs and tissue. Choice D is incorrect. Insomnia is not an expected assessment finding for iron deficiency anemia. These clients are typically frail and fatigued due to the decreased oxygen delivery to their organs and tissues. Additional Info ✓ Iron deficiency anemia is the most common worldwide ✓ Children may be affected and are sensitive to the neurocognitive effects ✓ All children are generally screened between 9 and 12 months ✓ Typical presentation is younger than three years old ✓ Manifestations associated with this anemia include pagophagia (craving for ice), pica, fatigue, mild pallor, and tachycardia

The nurse is caring for a 1-day old newborn client diagnosed with jaundice. Which of the following statements is true regarding jaundice in newborns? Select all that apply. Newborn jaundice is often caused by the baby's liver being immature. Phototherapy is a common treatment for newborn jaundice. Breastfeeding should be stopped if a newborn has jaundice. A jaundiced newborn's skin will appear yellow, starting with the face and progressing downwards. Severe, untreated newborn jaundice can lead to kernicterus, a type of brain damage.

Explanation Choices A, B, D, and E are correct. A is correct. Newborns often have jaundice due to an immature liver that cannot efficiently process bilirubin, a byproduct of the breakdown of red blood cells. This leads to the accumulation of bilirubin in the blood, causing the yellowish discoloration of the skin and whites of the eyes that is characteristic of jaundice (Maternal / Newborn Foundations of Maternal-Newborn & Women's Health Nursing, 7th Edition). B is correct. Phototherapy, or light therapy, is a standard treatment for newborn jaundice. The baby's skin absorbs the light, which changes the structure of the bilirubin molecules in a way that allows them to be excreted in the urine and stool (Maternal / Newborn Foundations of Maternal-Newborn & Women's Health Nursing, 7th Edition). D is correct. This is a characteristic presentation of newborn jaundice. The yellowish discoloration typically starts on the face. Then it spreads downwards to the chest, abdomen, arms, and legs as the bilirubin level increases (Maternal / Newborn Foundations of Maternal-Newborn & Women's Health Nursing, 7th Edition). E is correct. This is a critical point. If severe jaundice in a newborn is not treated, the high levels of bilirubin can lead to a condition called kernicterus, a form of brain damage that can result in irreversible neurological impairment (Maternal / Newborn Foundations of Maternal-Newborn & Women's Health Nursing, 7th Edition). Choice C is incorrect. C is incorrect. This is incorrect. Breastfeeding should not be stopped if a newborn has jaundice. Frequent feedings can help reduce the bilirubin level in the baby's body by promoting more frequent bowel movements, which helps to eliminate bilirubin from the body. However, in some cases, a temporary switch to formula may be recommended if the baby's bilirubin levels are very high (Maternal / Newborn Foundations of Maternal-Newborn & Women's Health Nursing, 7th Edition). Additional Info ✓Newborn jaundice is often a result of an immature liver, which cannot efficiently process bilirubin, a byproduct of the breakdown of red blood cells. ✓The condition is characterized by a yellowish discoloration of the skin and whites of the eyes, which starts from the face and

The nurse is teaching a parenting class on ways to prevent burn injuries in the home. Which of the following information should be included? Select all that apply. Protective guards should be installed in electrical outlets. Adjust your hot water heater to prevent scald burn injuries. If a liquid chemical burn occurs, do not rinse it with water Develop and practice a family escape plan in case of a fire. Pot handles should be turned toward the back of the stove.

Explanation Choices A, B, D, and E are correct. Protective guards should be placed in the outlet and furniture, if possible, should be placed in front of electrical outlets. This prevents the child from inserting objects into the outlet, preempting a serious electrical injury. The hot water heater should be adjusted to 49°C (120°F) or lower to prevent burn injuries. A fire escape plan should be developed for the home and practiced in the event of a fire. Pot handles should be placed on the back end of the stove with the handles turned toward the back. Choice C is incorrect. The initial treatment for a chemical burn is that it is irrigated with a copious amount of water. This should occur before touching the burn to prevent further injury to the individual rendering care. Additional Info Source : Archer Review Scald burns (e.g., hot water, grease, hot foods, etc.) are most common among young children, whereas flame burns are more prevalent among older children. The bathroom is where scald burns from tap water most often occur, and these injuries tend to be more severe, covering a larger portion of the body. Treatment is to immediately discontinue (or remove) the extremity from the heat source and run cool water for twenty minutes over the extremity. Chemical burns present special circumstances and require flushing with copious amounts of water. Irrigating the burn with water is essential before touching the chemical (even with gloves on). Neutralizing agents on the skin are contraindicated because a chemical reaction is initiated, and further injury may result

The nurse reviews cleft lip and cleft palate with a group of students. It would indicate effective teaching if the student states which are the following complications of both? Select all that apply. Ear infections Feeding difficulties Weight gain Speech delay Esophageal reflux

Explanation Choices A, B, and D are correct. When a child has a cleft lip and palate, the tissue and bone inside their mouth are not appropriately fused, meaning there is a space between their upper lip and palate. Ear infections will be a frequent complication for these patients due to the eustachian tube dysfunction, which connects the middle ear and the throat. Feeding issues are a common complication of cleft lip and cleft palate because it is harder for these infants to eat with the abnormality in their palate. The space in the roof of the mouth makes it very hard to suck and make a good seal around the bottle or nipple. Speech and language delays are common complications of cleft lip and palate. The roof of the mouth and lip have spaces that decrease muscle function and lead to delayed or abnormal speech. Eventually, many of these patients will require consultation with a speech-language pathologist. Choices C and E are incorrect. Weight gain is not a common complication of cleft lip and cleft palate. With these abnormalities, it is much more difficult for the infant to eat, and they commonly experience feeding issues— leading to weight loss or failure to thrive, not weight gain. Esophageal reflux is not consistent with this condition and is associated with gastroesophageal reflux disease (GERD).

The nurse is educating a group of students on the effects of corticosteroids. It would be appropriate for the nurse to identify the following adverse effects associated with corticosteroids. Select all that apply. Mood lability Immunosuppression Hypoglycemia Hyperkalemia Weight gain

Explanation Choices A, B, and E are correct. Glucocorticoids may cause potential mood changes such as overall mood lability. This may encompass a client feeling euphoric, agitated, depressed, and anxious. Exposure to long-term corticosteroids may cause an individual to develop leukopenia which may cause immunosuppression. Weight gain is a common finding with corticosteroid use because of the retention of sodium which causes the body to hold onto water. Choices C and D are incorrect. Hyperglycemia, not hypoglycemia, is a potential effect of corticosteroids. Corticosteroids cause an excessive discharge of glucose from the liver which raises glucose levels. Corticosteroids deplete potassium because of the effect of aldosterone (sodium retention; potassium elimination). Additional Info ✓ Corticosteroids may cause an array of adverse effects while they mitigate inflammation. ✓ This includes peptic ulcer disease, edema, hypokalemia, hyperglycemia, and hypernatremia. ✓ If not contraindicated, the client should be educated to maintain a low sodium and high potassium diet while taking corticosteroids.

The nurse is teaching a group of students about incident reports. Which of the following situations would require an incident report? A visitor Select all that apply. refusing to wear personal protective equipment (PPE). adjusting a client's infusion pump. requesting that their family member get pain medication. assisting their family member with brushing their teeth. stating that they fell while using the bathroom.

Explanation Choices A, B, and E are correct. Incident (sometimes termed occurrence or event) reporting is required when any activity deviates from the norm. Incident reporting may be completed for visitors. Events that warrant reporting would include the refusal to wear PPE, tampering with medical devices (such as adjusting an infusion pump), and reporting that they fell while using the bathroom. Choice C is incorrect. A visitor advocating for a client to receive pain medication does not require reporting Choice D is incorrect. A visitor assisting a client with oral hygiene does not require reporting. Additional Info ✓ Incident reporting, sometimes termed occurrence or event reporting, is a tool for mitigating future risks. Incident reporting should also be completed for events involving clients and visitors. Such events include: Verbal and physical displays of aggression Tampering with medical devices Reports of sexual or physical abuse by staff Adverse reaction to a blood transfusion Client elopement Damage (or loss) of client possessions Falls or injuries Injuries related to a medical device Complaints Medication and treatment errors Interfering with client care ✓ The incident should not be logged in the medical record/patient chart or nursing notes. ✓ Documentation, within the event reporting system, should be objective and factual and include what occurred, any injuries, if the provider was notified, care administered after the event, and any witnesses.

The nurse is caring for a client brought into the emergency department Item 1 of 1 Nurses' Notes 1545 - 60-year-old male arrived at the emergency department (ED) by EMS. The client was outside gardening and collapsed in his front yard. Upon arrival, the client could not communicate with staff, had obvious left-sided facial drooping, and was unable to perform any other tasks when instructed. The client could not even nod to yes/no questions. Pupils were reactive to light and 4 mm in size. Diagnostics 12-lead Electrocardiogram: Atrial fibrillation Vital Signs Temperature 98.0° F (37° C) Pulse 79/minute Respirations 14/minute Blood Pressure 174/108 mm Hg Oxygen saturation 95% on room air Select the three (3) prescriptions/orders the nurse should anticipate for this client Computed tomography scan of the brain Capillary blood glucose Lumbar puncture Arterial blood gas (ABG) Heparin by continuous IV infusion Nothing by mouth (NPO) status 500 mL of 0.9% saline

Explanation This client is experiencing manifestations of a cerebral vascular accident (CVA). Findings supporting the client having a CVA are the sudden onset of symptoms, inability to respond to questions (expressive aphasia), left-sided facial droop, and inability to perform purposeful movements when instructed. Finally, the client having atrial fibrillation on the electrocardiogram is likely the origin of the stroke. The nurse must transport the client for a CT scan of the brain as this will discern whether the stroke is hemorrhagic or ischemic. Capillary blood glucose is necessary to rule out hypoglycemia. Hypoglycemia (blood glucose less than 70 mg/dL) may manifest like stroke symptoms. The nurse should maintain nothing by mouth (NPO) status as the likelihood of aspiration is quite high, considering the client cannot follow commands. A lumbar puncture is unnecessary as the client is not displaying any signs or symptoms of an infection. ABGs are not indicated either, as the client shows no signs of hypoxia or metabolic concerns. Heparin by continuous infusion would not be plausible if this client was a candidate for tPA; thrombolytics would be given - not an anticoagulant. There is no clinical indication that this client needs ½ liter of saline as the client is hypertensive, a common finding during an ischemic stroke. Additional Info Strokes associated with atrial fibrillation have an abrupt onset. These ischemic strokes are caused by an embolus from the left atrial appendage. The FAST assessment (facial drooping, arm weakness or drift, slurred speech, time of last known well) is a rapid screening tool for a stroke. Other assessments include the Glasgow Coma Scale and the NIH Stroke Scale. The immediate priority for caring for a client with a suspected stroke is to assess their airway, breathing, and circulation and conduct a Glasgow Coma Scale assessment coupled with an NIH Stroke Scale. Pertinent laboratory work should be obtained, including capillary blood glucose, to rule out hypoglycemia. The client should be NPO until a bedside swallow evaluation can be completed.

The nurse is caring for a client who has been prescribed a 14-day course of prednisone. Which of the following statements, if made by the nurse, would be correct? Select all that apply. "This medication may make you gain weight." "It is best to take this medication in the morning with food." "If you have further pain, it is okay to take naproxen." "Your blood pressure may decrease while taking this medication." "Do not abruptly stop taking this medication."

Explanation Choices A, B, and E are correct. Prednisone is a corticosteroid and is indicated for various conditions, including exacerbations of rheumatoid arthritis. The medication potentiates aldosterone causing sodium and water retention, thereby allowing the client to gain weight. Steroids are best taken in the morning with food. Taking it with food decreases gastrointestinal upset. If the steroid is taken at nighttime, it may cause insomnia. The cessation of this drug should be tapered to avoid adrenal insufficiency. This medication should not be abruptly discontinued. Choices C and D are incorrect. Corticosteroids should not be combined with NSAIDs such as naproxen because that would hasten the risk of peptic ulcer disease. Blood pressure would increase because of fluid retention. Additional Info Corticosteroids may cause an array of adverse effects while they mitigate inflammation. This includes peptic ulcer disease, edema, hypokalemia, hyperglycemia, and hypernatremia. The client should be educated to maintain low sodium and high potassium diet while taking prednisone.

The nurse is teaching a continuing education course regarding vaccines and pregnancy. It would be appropriate for the nurse to state which vaccines are not recommended to be administered during pregnancy? Select all that apply. measles, mumps, and rubella (MMR) varicella hepatitis A inactivated Influenza tdap (Tetanus, Diphtheria, Pertussis) Human papillomavirus (HPV)

Explanation Choices A, B, and F are correct. These vaccines are contraindicated during pregnancy. If the client is scheduled to receive any of these vaccines, the nurse should inquire about the client's pregnancy status prior to vaccine administration. Choices C, D, and E are incorrect. Hepatitis A and B are safe to administer during pregnancy. Influenza vaccination is permitted during pregnancy but cannot be the live influenza vaccine (LAIV). The inactivated influenza vaccine is safe and approved for individuals who are pregnant. Tdap (Tetanus, Diphtheria, Pertussis) is permitted during pregnancy. Additional Info Vaccines either not recommended or contraindicated during pregnancy include: ✓ MMR ✓ Varicella ✓ Zoster ✓ HPV ✓ Polio ✓ Any live vaccine

The nurse is caring for a client receiving prescribed dexamethasone. Which of the following adverse reactions may occur? Select all that apply. Infection Hypotension Peripheral edema Hypoglycemia Weight loss Insomnia

Explanation Choices A, C and F are correct. Dexamethasone is a corticosteroid indicated in treating asthma exacerbations, rheumatoid arthritis, or pulmonary emphysema. Like all corticosteroids, prolonged exposure and higher doses increase the risk for adverse effects such as immunosuppression, fluid retention (peripheral edema), hypertension, irritability, and insomnia. Choices B, D, and E are incorrect. Corticosteroids lead to an increase in blood pressure, and the reasoning is multifactorial. The fluid retention likely increases blood pressure because of the increased blood volume. Corticosteroids increase the glucose discharge by the liver, raising serum glucose levels. Weight gain is commonly expected with treatment as increased appetite and fluid retention drive it. Additional Info ✓ Corticosteroids may cause an array of adverse effects while they mitigate inflammation ✓ Adverse effects include peptic ulcer disease, edema, hypokalemia, hyperglycemia, and hypernatremia ✓ Steroids should be dosed earlier in the day (if possible) to avoid effects such as insomnia ✓ Prolong exposure to corticosteroids may cause Cushing's syndrome

The school nurse is educating parents of children exposed to pediculosis capitis. Which of the following statements by the nurse would be appropriate to make? Select all that apply. Avoid sharing hats, caps, or scarves Dogs and cats need to be treated Nits are observable on the hair shaft You will need to repeat the treatment after seven to ten days Thoroughly vacuum carpets and upholstered furniture

Explanation Choices A, C, D, and E are correct. Good education regarding the prevention and treatment of pediculosis capitis (head lice) is essential. Lice do not fly or jump and are primarily spread by hats, scarves, combs, brushes, and other items used near the hair. White eggs (nits) are firmly attached to the hair shafts, and lice are small and grayish-tan, have no wings, and are visible to the naked eye. Treatment is through the application of permethrin 1% cream. This treatment must be repeated seven to ten days from the first treatment to ensure a cure. Central to the therapeutic management of this parasite is thoroughly vacuuming carpets, upholstered furniture, and laundering bed linens. Choice B is incorrect. Pets do not spread this parasite and do not have to be treated. Additional Info Pediculosis capitis is a parasite spread primarily by hygiene products and clothing articles ✓ White eggs (nits) firmly attached to the hair shafts. Lice are small and grayish-tan, have no wings, and are visible to the naked eye ✓ A classic manifestation of this infestation is pruritus, primarily behind the client's ears, occipital area, and nape of neck ✓ Lice do not fly or jump ✓ Treatment is 1% permethrin cream and retreating five to seven days after the initial treatment for a cure ✓ Home management is laundering linens, towels, hats, and scarves and preventing others from wearing these items ✓ Soak combs, brushes, and hair accessories in lice-killing products for 1 hour or in boiling water for 10 minutes ✓ Remove the nits from the child's hair with a metal nit comb daily

The nurse in the mental health unit is assessing a client with moderate anxiety. The nurse would anticipate which signs and symptoms to support this finding? Select all that apply. increased pulse feeling of impending doom reports of headache narrowing of the perceptual field inability to problem-solve or learn hyperventilation

Explanation Choices A, C, and D are correct. Moderate anxiety is characterized by a client experiencing - Narrowing of the perceptual field Slightly scattered thought process The client can problem-solve and learn, although not at an optimal level Somatic symptoms such as headache, urinary urgency, and muscle tension Sympathetic symptoms such as an increased pulse, respiratory rate, palpitations, voice tremors, and shaking Choices B, E, and F are incorrect. These manifestations align with severe anxiety. In severe anxiety, the client cannot engage in any problem solving and the somatic symptoms, such as headache and muscle tension, intensify The perceptual field is grossly narrowed, and the client may require directions because they may be dazed The thought process is not linear and significantly impaired The client may report signs of impending doom or dread Hyperventilation and tachycardia are commonly present Nursing care for severe anxiety is to remain with the client and be prepared to provide short, simple directions. Performing breathing exercises and providing reassurance. Prescriptive treatments such as hydroxyzine or diazepam may be warranted.

The client is diagnosed with acute pancreatitis. Which preventative intervention should the nurse implement to reduce the client's risk of developing a respiratory infection? Select all that apply. A. Assist the client to turn and reposition frequently B. Document the respiratory rate and oxygen saturation C. Place the client in a semi-fowlers position D. Encourage deep breathing and coughing. E. Obtain a prescription for prophylactic antibiotics

Explanation Choices A, C, and D are correct. Respiratory infections are common in acute pancreatitis due to retroperitoneal fluid pushing the diaphragm upwards and causing the client to take shallow abdominal breaths. Assisting the client to change positions frequently, encouraging deep breathing, coughing exercises, and positioning clients for maximum chest expansion would all be preventative interventions to reduce the risk of respiratory infection. Choices B and E are incorrect. While documentation would be indicated to recognize any changes or complications, it would not prevent respiratory complications associated with pancreatitis. Obtaining an antibiotic prescription is not reasonable and not directly a prevention strategy for respiratory complications. Giving antibiotics prophylactically may build resistance and cause unnecessary adverse reactions. Ambulation, cough, deep breathing, and repositioning are all practical and non-invasive ways to prevent respiratory complications. Additional Info ✓ Incentive spirometry can assist clients in their coughing and deep breathing exercises ✓ Early ambulation, if not contraindicated, can help prevent respiratory infections by getting the client upright and moving air more effectively

The nurse has provided discharge instructions to the parents of an infant with a newly applied Pavlik harness. Which of the following statements by the parents would indicate the need for additional teaching? Select all that apply. "I will remove the harness during feedings." "I will check for red areas under the straps and at the skin folds." "I will apply moisturizing lotion under the straps." "I will place the diaper over the straps." "I will gently massage the skin under the straps to stimulate circulation."

Explanation Choices A, C, and D are correct. These statements require follow-up because they are not correct. The harness should not be removed during feedings. The harness should be applied as directed by the prescriber, but generally, it is worn 24 hours a day. The healthcare provider should remove, adjust, and apply the harness. Lotion and powders should not be placed under the straps because that may cause dermatitis. The diaper is placed under the straps to avoid soiling the harness. Choices B and E are incorrect. Frequent skin assessments should occur by the parents because the straps may dig into the skin, causing breakdown. This should be reported to the PHCP for a potential adjustment of the straps. Gentle massage of the skin under the straps to stimulate circulation is permitted. Additional Info The Pavlik harness is a treatment method for developmental dysplasia of the hip (DDH) ✓ Treatment before 2 months often achieves the highest rate of success ✓ Treatment involves the application of a harness, casting, or surgery ✓ For the newborn to 6 months, the Pavlik harness may be applied ✓ This harness is applied, adjusted, and removed by the PHCP - not the parents ✓ The goal of the harness is to prevent hip extension and adduction ✓ Skin care is important while a client is wearing the harness ✓ Skin should be checked frequently for any reddened areas or overt skin breakdown ✓ Lotions and powders should not be used because of the potential for fungal dermatitis ✓ The diaper should be placed under the straps ✓ The infant should be dressed in loose, stretchy clothing ✓ If the straps get soiled, gentle soap and water via a washcloth may be used ✓ Provide sponge baths to the infant while leaving the harness in place ✓ Frequent follow-up appointments are necessary because the infant is growing

The nurse is assisting a client to choose food options appropriate for Celiac disease. Which food items would be appropriate to select? Select all that apply. Grilled chicken Wheat pasta Scrambled eggs Oatmeal Avocado

Explanation Choices A, C, and E are correct. Celiac disease is characterized by an individual's intolerance to gluten. Grilled chicken, scrambled eggs, and avocado are all examples of foods that have no gluten Choices B and D are incorrect. A client with Celiac disease should avoid sources of gluten such as barley, rye, oats, and wheat. Wheat pasta and oatmeal would not be appropriate selections for a client with gluten intolerance. Oats are not gluten-free as when they are transported they come into contact with other gluten-containing products. Additional Info Celiac disease, if untreated, may cause an individual abdominal pain, distention, vomiting, anemia, and diarrhea. A client should be thoroughly educated to avoid foods that contain gluten. Foods allowed include beef, chicken, pork, vegetables, fish, and eggs.

The nurse and two unlicensed assistive personnel (UAP) are preparing to reposition a client who requires log rolling. Which actions would be appropriate? Select all that apply. Place a small pillow between the client's knees. Places the client's arms at their side. Fanfold a drawsheet along the backside of the client. Instruct the client to laterally flex the neck during the turn. Roll the client as one unit in a smooth, continuous motion.

Explanation Choices A, C, and E are correct. These actions are appropriate during the process of log rolling a client. It is appropriate for a client who is to be log rolled to have a pillow placed between the client's knees to prevent tension on the spinal column and adduction of the hip. Fanning out a draw sheet under the client enables staff to have strong handles to grip without slipping. The purpose of log rolling a client is to move the client in one smooth, continuous motion to prevent twisting of the spinal column. Choices B and D are incorrect. To prevent a client from injuring their arms, the client should cross their arms across their chest during the repositioning. Instructing the client to laterally flex the neck would defeat the purpose of log rolling as this causes twisting of the spinal column. Additional Info Logrolling a client is utilized to keep the spinal column in straight alignment to prevent further injury. This turning technique is commonly used for clients with spinal cord injuries or who are recovering from neck, back, or spinal surgery. A minimum of three individuals is necessary to perform log rolling safely. The procedure of logrolling a client: Place a small pillow between the client's knees. Cross the client's arm on their chest. Position two nurses on the side where the client is to be turned and one nurse on the side where pillows are to be placed behind the patient's back. Fanfold drawsheet along the backside of the client. One nurse should grasp the drawsheet at the lower hips and thighs, and the other nurse grasping the drawsheet at the client's shoulders and lower back and roll the client as one unit in a smooth, continuous motion. The nurse on the opposite side of the bed places pillows along length of client for support. Gently lean the client as a unit back toward pillows for support.

The nurse is teaching a group of students about incident reports. Which of the following statements, if made by the student, would require further teaching? Select all that apply. "Reporting can only be completed if it is within one hour after the event." "Witnesses to an incident should be mentioned in the report." "A client eloping does not require an incident report." "A slip and fall by a client should be reported." "Incidents involving visitors do not have to be reported."

Explanation Choices A, C, and E are correct. These statements are false and require follow-up. Incident (sometimes termed occurrence or event) reporting is required when any type of activity deviates from the norm. This could include a fall, medication error, elopement, unplanned transfer of care, client complaint, or a delay in care. It is highly recommended that the event be reported after it occurs to ensure accuracy; however, event reporting should be completed as soon as possible, including the next day. No rule exists stating that reports must be completed within one hour after the actual event. Incidents involving visitors such as a fall, misconduct, complaint, or injury should be reported. Choices B and D are incorrect. These statements are true and do not require follow-up. The incident report should be completed with cohesion and based on facts. Supporting statements from witnesses and the identification of witnesses are appropriate to include in the report. Additional Info Incident (sometimes termed occurrence or event) reporting is a tool to mitigate future risks. Incident reporting should also be completed for events involving clients and visitors. Such events include: Verbal and physical displays of aggression Tampering with medical devices Reports of sexual or physical abuse by staff Adverse reaction to a blood transfusion Client elopement Damage (or loss) of client possessions Falls or injuries Injuries related to a medical device Complaints Medication and treatment errors Interfering with client care The incident should not be logged in the medical record or nursing notes. The documentation should be objective and factual and include what occurred, any injuries, if the provider was notified, care administered after the event, and any witnesses.

You are providing discharge teaching for a 3-year-old patient with CHF who has been prescribed digoxin. Which of the following medication instructions should be included in the discharge teaching? Select all that apply. Administer digoxin one hour before or two hours after meals. Mix the medication with milk or applesauce to ensure she drinks it all. If the child vomits after administering a dose then repeat the dose. Call the doctor if the child starts eating poorly and vomiting frequently. Administer at same time every day

Explanation Choices A, D and E are correct. This is the appropriate instruction to ensure proper absorption of digoxin. It is best to advise the parents to create a schedule and administer it at the same time each day, often before breakfast in the morning (Choice A). Poor feeding and frequent vomiting are signs of digoxin toxicity. This should be taught to the parents at discharge so that they can monitor their child for these symptoms and call the health care provider if they occur. This is the result of a timely lab test to determine the serum digoxin level and early treatment if toxicity has occurred (Choice D). . Administer the medication at the same time every day to maintain consistent blood levels (Choice E). Choice B is incorrect. This is not an appropriate action when administering digoxin. For the medication to be absorbed correctly, it must be taken on an empty stomach. Never administer digoxin with food. Choice C is incorrect. This is not an appropriate action when administering digoxin. A second dose should not be delivered, even if the child vomited after their first dose. Digoxin toxicity is severe and overdosing the child should always be avoided. Due to the potential toxicity, it is not advisable to administer a second dose, even if the child vomited. NCSBN Client Need: Topic: Physiological Integrity, Subtopic: Physiological Adaptation, Cardiovascular Additional Info ✓ Digoxin is usually given orally, and the child should be instructed to swallow the medication rather than chewing or crushing it. If a child has difficulty swallowing the medication, the healthcare provider may consider using an alternative form of administration, such as a liquid or crushed tablet mixed with food. ✓ Children taking digoxin should be monitored for signs of toxicity, which can include nausea, vomiting, headache, visual changes, or irregular heart rhythm. Healthcare providers should also monitor the child's heart rate, rhythm, and electrolyte levels, which can affect the effectiveness and safety of digoxin. ✓ Parents and caregivers should be educated on the proper administration of digoxin, including the importance of giving the medication at the same time each day and not missing doses. They should also be in

The nurse is conducting a health screening at a local health fair. Which of the following should the nurse recognize as risk factors for developing colorectal cancer? Select all that apply. Ulcerative colitis Body Mass Index (BMI) of 21 Human Immunodeficiency Virus (HIV) infection Low-fiber diet Excessive alcohol consumption African-American ethnicity

Explanation Choices A, D, E, and F are correct. Inflammatory bowel disease (especially ulcerative colitis) is a non-modifiable risk factor that may cause cellular damage and hasten the risk of colorectal cancer. A diet low in fiber is a modifiable risk factor for colon cancer. Encourage the client to increase fiber intake and decrease red meat. Excessive alcohol intake is a modifiable risk factor for colorectal cancer. African American ethnicity is a non-modifiable risk factor for colorectal cancer. Choice B is incorrect. A BMI of 21 is optimal and is not a risk factor. Obesity is a modifiable risk factor for colorectal cancer. Obesity is defined as a Body Mass Index (BMI) ≥ 30 kg/m2. Choice C is incorrect. HIV is a risk factor for many malignancies such as testicular cancer, but not colorectal cancers. Since rates of colorectal cancer are similar between people with and without HIV, existing screening guidelines are sufficient for people with HIV. Another virus called human papillomavirus (HPV) has been implicated in colorectal cancers. Additional Info ✓ Risk factors for colorectal cancer are divided into modifiable and non-modifiable types. ✓ Modifiable risk factors are usually behavioral factors that can increase a person's risk of cancer. In theory, these risk factors can be modified with interventions. ✓ Non-modifiable risk factors are those that can not be changed. Awareness of the client's risk factors will help the healthcare provider prescribe personalized lifestyle and cancer screening recommendations. ✓ The gold standard of colorectal cancer prevention is a colonoscopy that should begin as early as age 45 (USPTF new guidelines, 2021).

The nurse is caring for a client with a tracheostomy receiving oxygen via tracheostomy collar. The nurse should plan to Select all that apply. Plan to suction the tracheostomy every two to four hours Ensure that the oxygen is humidified Instill normal saline down the tracheostomy immediately before suctioning Suction the tracheostomy for a maximum of three passes Apply suction as the catheter is inserted into the tracheostomy

Explanation Choices B and D are correct. When caring for a client with a tracheostomy, suctioning should be performed only when clinically indicated. Indications for suctioning the client include tachypnea, rhonchi in the lung fields, and decreasing oxygen saturation. When suctioning a tracheostomy, the nurse should use a sterile technique, and a maximum of three passes should be completed. The oxygen must be warm and humidified for a client receiving oxygen via a trach collar. If it is not appropriately warmed or humidified, tracheal damage may occur. The humification assists with the passage of the secretions. Choices A, C, and E are incorrect. Instilling saline down the tracheostomy prior to suctioning has shown little benefit and may be harmful because it could lead to pneumonia. When the client's tracheostomy is suctioned, it should be suctioned as the catheter is withdrawn for 10-15 seconds. The suctioning should be continuous so secretions are not dropped back down the airway. Suctioning should not be scheduled; instead, it should be based on the client's respiratory assessment.

The RN is the only RN in the assisted care facility on a busy evening shift. Of the following tasks, which ones can be safely delegated to an experienced LPN/LVN? Select all that apply. Completing an admission assessment on a new patient Administering routine oral medications to stable patients. Removal of a urinary catheter Completing a dressing change Administering an initial dose of a new medication to a patient.

Explanation Choices B, C, and D are correct. B is correct. LPN/LVN training allows nurses to do tasks with the most predictable outcomes, including passing oral medication to stable clients. LPNs/LVNs can administer routine medications under the RN's supervision. C is correct. LPN/LVN training allows nurses to do tasks with the most predictable outcomes, such as removing a Foley catheter. D is correct. LPN/LVN training allows nurses to do tasks with the most predictable outcomes, such as completing routine dressing changes. Choices A and E are incorrect. A is incorrect. The LPN/LVN role does NOT include admission or focused assessments, initial patient education, or any activity that requires critical nursing decision-making. E is incorrect. Administering an initial dose of a new medication to a patient is a task typically not delegated, as it often requires the assessment and critical thinking skills of an RN. An RN must closely monitor the patient's reaction to the new medication. Additional Info It's crucial for LPNs/LVNs and their employers to familiarize themselves with the specific nursing practice act and regulations in their state. State Boards of Nursing are typically the best resource for this information. The scope of practice for Licensed Practical Nurses (LPNs) or Licensed Vocational Nurses (LVNs) can vary by state, but commonly includes the following: ✓Providing basic patient care, including monitoring vital signs, bathing, and dressing. ✓Administering injections and medications (excluding certain high-risk medications or routes). ✓Performing routine laboratory tests and sample collections. ✓Assisting with wound care and dressing changes. ✓Assisting patients with activities of daily living (ADLs). ✓Reporting changes in patient status to RNs or physicians. ✓Providing patient and family education under the supervision of an RN. However, some tasks are usually outside the LPN/LVN scope of practice: ✓Initial patient assessments, which are usually done by RNs. ✓Administering certain types of medications, such as those given intravenously (although this can vary by state). ✓Developing and modifying nursing care plans. ✓Performing certain complex procedures. ✓Working independently without supervision

The nurse is caring for a client in the mental health unit experiencing psychosis Item 1 of 1 Nurses' Notes 1300 - Client was banging their food tray and shouting at other clients. De-escalated the situation by escorting the client back to their room. Once back to their room, the client kept shouting 'they are after me!' at the roommate. The client stopped the shouting but resumed several minutes later. The client refused the scheduled by-mouth (PO) olanzapine, stating, 'they don't want me to take that. Medications Olanzapine 10 mg PO Daily Medical History Schizophrenia Vitamin D deficiency Hyperlipidemia Select two (2) actions the nurse should take Provide therapeutic touch Limit interaction with the client Place the client in seclusion Ask if the client hears any voices Crush the olanzapine in the client's food Reassign the client to a private room

Explanation This client is experiencing overt paranoia, and the nurse should ask if the client hears any voices. Auditory hallucinations are the most common perceptual disturbance associated with schizophrenia and may contribute to a client's paranoia. If auditory hallucinations are evident, the nurse can intervene by exploring the content of the hallucination and reorientating the client. Reassigning the client to a private room is appropriate. The client is in the mental health unit, and milieu therapy provides all clients with an organized, structured, and safe environment. The client's condition is disrupting this type of therapy, and for the safety of the other clients, the client should be reassigned to a private room close to the nursing station. Therapeutic touch is not recommended because the client experiencing paranoia may misinterpret this action. Interaction should not be limited because this would hinder the therapeutic relationship. The client's behavior also warrants increased monitoring. Seclusion is not recommended because the client is not exhibiting any violence (self-directed or towards others). Since the client has refused the medication, crushing olanzapine and placing it in the client's food is unethical and not permitted. Additional Info A client experiencing psychosis has a detachment from reality. Psychosis is often described as the fever of schizophrenia. Key Interventions for Psychosis and Paranoia: Maintain a safe environment Assess the client for suicidal and homicidal ideations Establish a therapeutic rapport Avoid therapeutic touch and whispering in the client's presence Focus on reality-based topics and frequently reorientate the client.

The nurse is assessing a client who was admitted four hours ago with hypomagnesemia. Which of the following findings should the nurse recognize as a common cause of hypomagnesemia? Select all that apply. Renal failure Alcoholism Anorexia nervosa Diarrhea Hypothyroidism

Explanation Choices B, C, and D are correct. Alcoholism causes diuresis, which lowers serum magnesium levels. Additionally, chronic alcoholism impairs the absorption of magnesium. Anorexia nervosa is a psychiatric illness where the individual eats very few calories that causes electrolyte disturbances such as low potassium, magnesium, and sodium. All of which may be life-threatening. Diarrhea causes a depletion of all electrolytes, which would appropriately explain the low magnesium levels. Choices A and E are incorrect. Renal failure can cause hypermagnesemia because the process that keeps the magnesium levels in the body at normal levels does not work properly in people with kidney dysfunction. Hypothyroidism is a risk factor for hypermagnesemia because it causes magnesium reabsorption in the renal tubules. Additional Info ✓ The normal level of magnesium is 1.5-2.5 mEq/L ✓ Food sources rich in magnesium include pumpkin seeds, almonds, dark leafy vegetables, soybeans, and dried figs ✓ Causes of hypomagnesemia include alcoholism, diarrhea, and diuretics ✓ Causes of hypermagnesemia include renal failure, certain antacids, excessive intake, lithium therapy, adrenal insufficiency, and hypothyroidism

The nurse is preparing to provide care for a client with disseminated herpes zoster. The nurse plans to don which personal protective equipment (PPE)? Select all that apply. goggles gown gloves shoe covers n95 respirator Surgical face mask

Explanation Choices B, C, and E are correct. When varicella zoster is disseminated, it can be transmitted through airborne means and by direct contact with the lesions. The isolation required is contact + airborne. This means the nurse should wear an N95 respirator, high-efficiency particulate air filter respirator, gown, and gloves. Herpes zoster, also known as shingles, is caused by the reactivation of the varicella-zoster virus (VZV), the same virus that causes varicella (chickenpox). Primary infection with VZV causes varicella. Once the illness resolves, the virus remains latent in the dorsal root ganglia. VZV can be reactive later in a person's life and create a painful, maculopapular rash called herpes zoster. When herpes zoster (varicella-zoster) (shingles) is localized and can be covered, standard precautions are implemented until all of the lesions have crusted over. When herpes zoster (varicella-zoster) (shingles) is disseminated, airborne + contact precautions are implemented for the duration of the illness. Choices A and D are incorrect. Goggles and shoe covers are not needed for airborne or contact precautions. Choice F is incorrect. A surgical face mask filters only large particles and will not protect against herpes zoster.

The nurse is caring for a client in the second stage of labor and is experiencing labor dystocia. Which of the following options can the physician perform to facilitate delivery? Select all that apply. Amniotomy Forceps assisted delivery External version Vacuum assisted delivery Episiotomy

Explanation Choices B, D, and E are correct. Forceps are tools used to help pull on the head of the baby to assist with the delivery. Vacuum-assisted delivery is a method where suction is applied to the head of the baby and pulled while the mother pushes. Episiotomy is a surgical incision in the perineum to widen the vaginal opening and facilitate fetal head delivery. These methods may be used for labor dystocial during the second (pushing stage) stage. Choices A and C are incorrect. An amniotomy uses a hook or finger to break the amniotic sac. This helps to induce labor but does not assist in the delivery of the head of the fetus. This client is in the second stage of labor (pushing stage), and rupturing of the amniotic fluid sac should have already occurred. The external version is a technique used when the baby is not in an appropriate position for vaginal delivery. The external cephalic version turns a fetus from a breech position or side-lying (transverse) position into a more favorable head-down (vertex) position to help prepare the baby for a vaginal delivery. The external version is typically done before the labor begins, often around 37 weeks. These approaches would be completed before the second stage of labor. Additional Info ✓ The second stage of labor is known as the pushing stage ✓ It begins with complete (10 cm) dilation and full (100%) effacement of the cervix and ends with the birth of the baby ✓ Contractions are strong and about 2 to 3 minutes apart, lasting 40 to 60 seconds Dystocia is a term used to describe difficult or prolonged labor, typically due to an obstruction in the birth canal or other issues that interfere with the progress of labor. The following are some common causes of dystocia: Malpresentation of the fetus: When the fetus is not positioned correctly, such as being in a breech position, labor may be prolonged or difficult. Fetal macrosomia: When the fetus is larger than average, it may not fit easily through the birth canal, leading to dystocia. Pelvic abnormalities: Certain pelvic abnormalities can make it difficult for the fetus to pass through the birth canal. Inefficient uterine contractions: Weak or infrequent contractions may prolong labor and lead to dystocia. Mater

Select the stage of development, according to Eric Erickson that is accurately paired with its developmental task. Select all that apply. Infant: Egocentricity versus Integration. Toddler: Autonomy versus Mistrust. Adolescent: Identity versus Role Confusion. Elder years: Integrity versus Despair. Preschool Child: Initiative versus Guilt.

Explanation Choices C, D, and E are correct. The stages of development, according to Eric Erickson, that are accurately paired with their developmental task are: Adolescent: Identity versus Role Confusion (Choice C), Elder years: Integrity versus Despair (Choice D), and Preschool Child: Initiative versus Guilt (Choice E). The other developmental tasks, according to Erik Erikson, are: Infant: Trust versus Mistrust Toddler: Autonomy versus Shame and Doubt School-Aged Child: Industry versus Inferiority Young Adult: Intimacy versus Isolation Middle Years: Generativity versus Stagnation Choices A and B are incorrect. The infant is not challenged with Egocentricity versus Integration; they are challenged with Trust versus Mistrust. The infant stage revolves around building trust. The toddler is not challenged with Autonomy versus Mistrust; they are challenged with Autonomy versus Shame and Doubt. Toddlers are focused on developing a sense of independence and a sense of personal control over physical skills.

The home health nurse visits a client with chronic diabetes insipidus Nurses' Note 1415 - Home health visit completed because the client was admitted to the hospital twice in the past six weeks for treatment nonadherence related to diabetes insipidus. Extensive teaching provided and reviewed education on prescribed desmopressin intranasal, maintenance of fluids, daily weight, intake and output, and when to seek emergency care. Which four (4) client statements would indicate a correct understanding of the teaching? "I should limit the amount of fluids that I drink after 5:00 PM." "I will need to weigh myself at the same time every day." "I should put both doses of the desmopressin in one nostril." "I need to keep a log of my fluid intake and urine output." "I may need an additional dose if I keep urinating a lot." "If I develop confusion with this medication, I should call 911."

Explanation Clients with permanent diabetes insipidus will need lifelong treatment involving desmopressin, which can be dosed intranasally or by mouth. Diabetes insipidus requires teaching encompassing the prescribed therapy, fluid maintenance, daily weight, intake and output, and when the client should seek emergency care. It will indicate correct understanding if the client verbalizes that they need to weigh themselves at the same time of day while wearing a similar amount of clothing. Additionally, the client should log their fluid intake and urine output as they should drink fluids in an amount equal to urine output. Further, a client may take an additional dose of their prescribed desmopressin if they keep experiencing polyuria and polydipsia. An adverse effect of desmopressin is that it may cause water toxicity (persistent headache, acute confusion, nausea, vomiting) a medical emergency. The client does not need to restrict their fluid intake after 5 p.m. This would be appropriate teaching for a client receiving a diuretic. The client with DI should not be on fluid restrictions. Additional Info Source : Archer Review Diabetes insipidus may be caused by an insufficient antidiuretic hormone from the posterior pituitary or the kidney's response to antidiuretic hormone. Clinical Manifestations of Diabetes Insipidus: Symptoms of dehydration Polyuria (4 to 30 L) a day Excessive thirst Low urine specific gravity A significant complication of DI is hypovolemic shock


Kaugnay na mga set ng pag-aaral

Chapter 6 Basic Cognitive Functions: Information Processing, Attention, and Memory (adult)

View Set

mass communication- Audio; Radio and Podcast

View Set

Chapter 11 Testbank: Respiratory & Lungs

View Set